65

1. Пояснительная запискаjar.edunoskol.ru/images/dokument/RAB_PROG/geometria_coo.pdf · Стандарт основного общего образования

Embed Size (px)

Citation preview

Page 1: 1. Пояснительная запискаjar.edunoskol.ru/images/dokument/RAB_PROG/geometria_coo.pdf · Стандарт основного общего образования
Page 2: 1. Пояснительная запискаjar.edunoskol.ru/images/dokument/RAB_PROG/geometria_coo.pdf · Стандарт основного общего образования

1. Пояснительная записка

Рабочая программа учебного предмета «Геометрия 10-11» составлена на основе

следующих нормативных документов:

1. Стандарт основного общего образования по математике. (Стандарт основного

общего образования по математике //Математика в школе. – 2004г., №4).

2. Авторская программа (Программы общеобразовательных учреждений.

Геометрия. 10-11 классы. – М. Просвещение, 2009. Составитель:

БурмистроваТ.А.)

Данная программа изучения геометрии ориентирована на изучение математики в

старшей школе на базовом уровне и направлена на достижение следующих целей:

формирование представлений о математике как универсальном языке науки,

средстве моделирования процессов, об идеях и методах математики;

развития логического мышления, алгоритмической культуры, критичности

мышления на уровне, необходимом для обучения в высшей школе по

соответствующей специальности;

овладение математическими знаниями и умениями, необходимыми в

повседневной жизни, для изучения школьных естественнонаучных

дисциплин на базовом уровне;

воспитание средствами математики культуры личности, отношения к

математике как к части общечеловеческой культуры через знакомство с

историей развития математики, понимание значимости математики для

общественного процесса.

При изучении курса на базовом уровне решаются следующие задачи:

познакомить учащихся с содержанием курса стереометрии, с основными

понятиями и аксиомами, принятыми в данном курсе, вывести первые

следствия из аксиом, дать представления о геометрических телах и их

поверхностях, об изображении пространственных фигур на чертеже, о

прикладном значении геометрии;

сформировать представления учащихся о возможных случаях взаимного

расположения двух прямых в пространстве, прямой и плоскости; изучить

свойства и признаки параллельности прямых и плоскостей;

ввести понятие перпендикулярности прямых и плоскостей, изучить

признаки перпендикулярности прямой и плоскости, двух плоскостей, ввести

основные метрические понятия: расстояние от точки до плоскости,

расстояние между параллельными плоскостями, между параллельными

прямой и плоскостью расстояние между скрещивающимися прямыми, угол

между двумя плоскостями, изучить свойства прямоугольного

параллелепипеда;

познакомить учащихся с основными видами многогранников, с

правильными многогранниками и элементами их симметрии.

Программа составлена на основе Базисного учебного плана; на предмет

«Геометрия 10-11» отводится 138 часов (2 часа в неделю).

Page 3: 1. Пояснительная запискаjar.edunoskol.ru/images/dokument/RAB_PROG/geometria_coo.pdf · Стандарт основного общего образования

Учебно-методический комплект для изучения курса Геометрии в 10-11 классах

состоит из следующих компонентов:

Программы общеобразовательных учреждений. Геометрия. 10 – 11 классы,

- М.: Просвещение, 2009. Составитель: Бурмистрова Т.А .

Тематическое планирование по математике для 10 – 11 классов: пособие

для учителя / Составитель: Т.А. Бурмистрова - М.: Просвещение, 2007.

Геометрия. 10 - 11: учеб. для общеобразоват. учреждений / Л.С. Атанасян

и др./ М.: Просвещение, 2006.

Зив Б.Г. Геометрия: дидактические материалы для 11 класса. М.:

Просвещение, 2012.

2. Требования к уровню подготовки учащихся

В результате изучения геометрии на базовом уровне ученик должен

знать/понимать

значение математической науки для решения задач, возникающих в теории и

практике; широту и в то же время ограниченность применения математических

методов к анализу и исследованию процессов и явлений в природе и обществе;

значение практики и вопросов, возникающих в самой математике для формирования

и развития математической науки; историю развития понятия числа, создания

математического анализа, возникновения и развития геометрии;

универсальный характер законов логики математических рассуждений, их

применимость во всех областях человеческой деятельности;

уметь

распознавать на чертежах и моделях пространственные формы; соотносить

трехмерные объекты с их описаниями, изображениями;

описывать взаимное расположение прямых и плоскостей в пространстве,

аргументировать свои суждения об этом расположении;

анализировать в простейших случаях взаимное расположение объектов в

пространстве;

изображать основные многогранники и круглые тела; выполнять чертежи по

условиям задач;

строить простейшие сечения куба, призмы, пирамиды;

решать планиметрические и простейшие стереометрические задачи на нахождение

геометрических величин (длин, углов, площадей, объемов);

использовать при решении стереометрических задач планиметрические факты и

методы;

проводить доказательные рассуждения в ходе решения задач;

использовать приобретенные знания и умения в практической деятельности и

повседневной жизни для:

исследования (моделирования) несложных практических ситуаций на основе

изученных формул и свойств фигур;

вычисления объемов и площадей поверхностей пространственных тел при решении

практических задач, используя при необходимости справочники и вычислительные

устройства.

В результате изучения геометрии в 10-11 классах ученик должен знать и уметь:

Page 4: 1. Пояснительная запискаjar.edunoskol.ru/images/dokument/RAB_PROG/geometria_coo.pdf · Стандарт основного общего образования

соотносить плоские геометрические фигуры и трехмерные объекты с их

описаниями, чертежами, изображениями; различать и анализировать взаимное

расположение фигур;

изображать геометрические фигуры и тела, выполнять чертеж по условию задачи;

решать геометрические задачи, опираясь на изученные свойства планиметрических

и стереометрических фигур и отношений между ними, применяя алгебраический и

тригонометрический аппарат;

проводить доказательные рассуждения при решении задач, доказывать основные

теоремы курса;

вычислять линейные элементы и углы в пространственных конфигурациях,

площади поверхностей пространственных тел и их простейших комбинаций;

применять координатно-векторный метод для вычисления отношений, расстояний

и углов;

строить сечения многогранников;

Page 5: 1. Пояснительная запискаjar.edunoskol.ru/images/dokument/RAB_PROG/geometria_coo.pdf · Стандарт основного общего образования

3. Календарно-тематическое планирование

10 класс

урока

Пункт

учеб-

ника

Содержание учебного материала Кол-во

часов

Дата проведения Примечание

плани-

руемая

факти-

ческая

Введение 3

1

2

3

п. 1-3 Предмет стереометрии.

Основные понятия и аксиомы стереометрии.

Первые следствия из аксиом

1

1

1

http://belclass.net

Глава I Параллельность прямых и плоскостей 20 http://fcior.edu.ru

4

5

6

7

п. 4-6 Параллельность прямых. Входной тест

Параллельность прямой и плоскости.

Параллельность трѐх прямых.

Параллельность прямой и плоскости.

1

1

1

1

Демоверсия ЕГЭ-2014

8

9

10

11

п. 7-9 Скрещивающиеся прямые.

Углы с сонаправленными сторонами.

Угол между прямыми.

Решение задач по теме «Параллельность прямых и плоскостей»

1

1

1

1

Математика.

Подготовка к ЕГЭ-2015. Под

редакцией

Ф.Ф. Лысенко,

С.Ю. Кулабухова

12 п. 4-9 Контрольная работа № 1 по теме «Параллельность прямых и

плоскостей» 1

13-14 п. 10-11 Анализ контрольной работы.

Параллельность плоскостей

2 Диагностическая работа №1 (СтатГрад)

15-19 п. 12-14 Тетраэдр и параллелепипед 5 В5

20 п. 10-14 Контрольная работа № 2 по теме «Параллельность прямых и

плоскостей» 1

21

22

п. 4-14 Анализ контрольной работы.

Решение задач по теме «Построение сечений»

Повторение по теме «Параллельность прямых и

плоскостей»

1

1

В8

23 п. 4-14 Зачѐт № 1 по теме «Параллельность прямых и

плоскостей»

1

Page 6: 1. Пояснительная запискаjar.edunoskol.ru/images/dokument/RAB_PROG/geometria_coo.pdf · Стандарт основного общего образования

Глава II

Перпендикулярность прямых и плоскостей

19

24-28 п. 15-18 Перпендикулярность прямой и плоскости

5 Демоверсия ЕГЭ-2014

29-31

32-34

п. 19-21 Перпендикуляр и наклонная.

Промежуточное тестирование

Угол между прямой и плоскостью

3

3

В10

35

36-38

п. 22-24 Двугранный угол

Перпендикулярность плоскостей

1

3

Тренировочная Работа

№1(СтатГрад)

39 п. 15-24 Контрольная работа № 3 по теме «Перпендикулярность

прямых и плоскостей»

1

40-41 п. 15-24 Анализ контрольной работы. Повторение по теме

«Перпендикулярность прямых и плоскостей»

2

42 п. 15-24 Зачѐт № 2 по теме «Перпендикулярность прямых и

плоскостей»

1

Глава III

Многогранники

20

43-47

п. 25-27

Понятие многогранника. Призма

5

Page 7: 1. Пояснительная запискаjar.edunoskol.ru/images/dokument/RAB_PROG/geometria_coo.pdf · Стандарт основного общего образования

48-52 п. 28-30 Пирамида

5 В13

53-57 п. 31-33 Правильные многогранники

5 В5

58 п. 25-33 Контрольная работа № 4 по теме «Многогранники» 1 В8

59-61 п. 25-33 Анализ контрольной работы. Повторение по теме

«Многогранники»

3 В10

62 п. 25-33 Зачѐт № 3 по теме «Многогранники»

1 С2

Итоговое повторение

8

63-64 п. 4-14 Повторение по теме «Параллельность прямых и

плоскостей»

2 С4

65-66 п. 15-24 Повторение по теме «Перпендикулярность прямых и

плоскостей»

2 СтатГрад

67-70 п. 25-33 Повторение по теме «Многогранники»

Итоговый тест в форме ЕГЭ

4 Демо ЕГЭ-2015

11 класс

Page 8: 1. Пояснительная запискаjar.edunoskol.ru/images/dokument/RAB_PROG/geometria_coo.pdf · Стандарт основного общего образования

урока

Пункт

учеб-

ника

Содержание учебного материала Кол-во

часов

Дата проведения Примечание

(ЦОР, подготовка к ЕГЭ)

плани-

руемая

факти-

ческая

Глава IV Векторы в пространстве 6

http://belclass.net

1 п. 34-35

Понятие вектора в пространстве 1 http://mathege.ru

2

3

п. 36-38 Сложение и вычитание векторов.

Умножение вектора на число

1

1

4-5 п. 39-41 Компланарные векторы.

Входной тест

2

6 п. 34-41 Зачѐт № 1 по теме «Векторы в пространстве»

1

Глава V Метод координат в пространстве

15

7

8

9

10-12

п. 42-45 Прямоугольная система координат в пространстве

Координаты вектора

Связь между координатами векторов и точек

Простейшие задачи в координатах

1

1

1

3

http://fcior.edu.ru/card/4050/koordinaty-

vektora-clozhenie-vektorov-i-umnozhenie-

vektora-na-chislo.html

13

14

15-16

17

18

п. 46-48

п. 49-52

Угол между векторами

Скалярное произведение векторов

Вычисление углов между прямыми и плоскостями

Центральная симметрия. Осевая симметрия.

Зеркальная симметрия. Параллельный перенос.

1

1

2

1

1

Математика.

Подготовка к ЕГЭ-2015.

Под редакцией

Ф.Ф. Лысенко,

С.Ю. Кулабухова

19

п. 42-48 Контрольная работа № 1 по теме «Метод координат в

пространстве»

1

20 Анализ контрольной работы. Повторение по теме

«Векторы в пространстве».

1 http://fcior.edu.ru/card/28114/primery-

dvizheniy-figur-simmetriya-figur.html

21 п. 34-52 Зачѐт № 2 по теме «Метод координат в пространстве»

1

Page 9: 1. Пояснительная запискаjar.edunoskol.ru/images/dokument/RAB_PROG/geometria_coo.pdf · Стандарт основного общего образования

Глава VI Цилиндр, конус и шар

16

22-23

24

п. 53-54 Цилиндр. Площадь поверхности цилиндра

Площадь поверхности цилиндра

2

1

В5

25

26

27-28

п. 55-57 Понятие конуса.

Площадь поверхности конуса.

Усечѐнный конус.

1

1

2

Диагностическая работа №1 (СтатГрад)

29

30

31

32-34

п. 58-62 Сфера и шар. Уравнение сферы.

Взаимное расположение сферы и плоскости.

Касательная плоскость к сфере. Площадь сферы.

Разные задачи на цилиндр, конус и шар

1

1

1

3

С2

35

п. 53-62 Контрольная работа № 2 «Цилиндр, конус и шар»

1

36 п. 53-62 Анализ контрольной работы. Повторение основных

вопросов курса.

1 В8

37 п. 53-62 Зачѐт № 3 по теме «Цилиндр, конус и шар»

1

Глава VII Объемы тел 17

38-40 п. 63-64 Объѐм прямоугольного параллелепипеда

3 Демоверсия ЕГЭ-2014

41-42 п. 65-66 Объѐм прямой призмы и цилиндра 2 В10

43

44

45

46-47

п. 67-70 Вычисление объѐмов тел

Объем наклонной призмы.

Объем пирамиды.

Объем конуса

1

1

1

2

Тренировочная Работа

№!1(СтатГрад)

48

49-50

51

п. 71-73

Объем шара

Объѐм шарового сегмента.

1

2

Page 10: 1. Пояснительная запискаjar.edunoskol.ru/images/dokument/RAB_PROG/geometria_coo.pdf · Стандарт основного общего образования

Площадь сферы 1

52 п. 63-73 Контрольная работа № 3 по теме «Объемы тел» 1

53

54

п. 63-73

п. 63-73

Анализ контрольной работы.

Повторение по теме «Объемы тел»

Зачет № 4 по теме «Объемы тел»

1

1

В13

Итоговое повторение

14

55-56 п. 4-14

Повторение по теме «Параллельность прямых и

плоскостей»

2 В5

57-58 п. 15-24

Повторение по теме «Перпендикулярность прямых и

плоскостей»

2 В8

59-60 п. 34-52 Повторение по теме «Векторы в пространстве»

2 В10

61-62 Повторение по теме «Многогранники и тела вращения» 2 С2

63-64 п. 25-33

п. 53-62

Повторение по теме «Объемы тел» 2 С4

65-66 п. 63-73 Решение задач в формате ЕГЭ

2 Демо ЕГЭ-2015

67-68 Итоговый тест в формате ЕГЭ.

Анализ итогового теста

2 СтатГрад

Page 11: 1. Пояснительная запискаjar.edunoskol.ru/images/dokument/RAB_PROG/geometria_coo.pdf · Стандарт основного общего образования

4. Содержание программы учебного курса

10 класс (2 ч в неделю, всего 70 ч)

1. Введение (аксиомы стереометрии и их следствия). (3 ч).

Представление раздела геометрии – стереометрии. Основные понятия стереометрии.

Аксиомы стереометрии и их следствия. Многогранники: куб, параллелепипед,

прямоугольный параллелепипед, призма, прямая призма, правильная призма, пирамида,

правильная пирамида. Моделирование многогранников из разверток и с помощью

геометрического конструктора.

Цель: ознакомить учащихся с основными свойствами и способами задания плоскости на

базе групп аксиом стереометрии и их следствий.

О с н о в н а я ц е л ь – сформировать представления учащихся об основных понятиях и

аксиомах стереометрии, познакомить с основными пространственными фигурами и

моделированием многогранников.

Особенностью учебника является раннее введение основных пространственных фигур, в

том числе, многогранников. Даются несколько способов изготовления моделей

многогранников из разверток и геометрического конструктора. Моделирование

многогранников служит важным фактором развития пространственных представлений

учащихся.

2. Параллельность прямых и плоскостей. (19 ч).

Пересекающиеся, параллельные и скрещивающиеся прямые в пространстве.

Классификация взаимного расположения двух прямых в пространстве. Признак

скрещивающихся прямых. Параллельность прямой и плоскости в пространстве.

Классификация взаимного расположения прямой и плоскости. Признак параллельности

прямой и плоскости. Параллельность двух плоскостей. Классификация взаимного

расположения двух плоскостей. Признак параллельности двух плоскостей. Признаки

параллельности двух прямых в пространстве.

Цель: дать учащимся систематические знания о параллельности прямых и плоскостей в

пространстве.

О с н о в н а я ц е л ь – сформировать представления учащихся о понятии параллельности

и о взаимном расположении прямых и плоскостей в пространстве, систематически

изучить свойства параллельных прямых и плоскостей, познакомить с понятиями вектора,

параллельного переноса, параллельного проектирования и научить изображать

пространственные фигуры на плоскости в параллельной проекции.

В данной теме обобщаются известные из планиметрии сведения о параллельных прямых.

Большую помощь при иллюстрации свойств параллельности и при решении задач могут

оказать модели многогранников.

Здесь же учащиеся знакомятся с методом изображения пространственных фигур,

основанном на параллельном проектировании, получают необходимые практические

навыки по изображению пространственных фигур на плоскости. Для углубленного

изучения могут служить задачи на построение сечений многогранников плоскостью.

3. Перпендикулярность прямых и плоскостей. (19ч).

Угол между прямыми в пространстве. Перпендикулярность прямых. Перпендикулярность

прямой и плоскости. Признак перпендикулярности прямой и плоскости. Ортогональное

проектирование. Перпендикуляр и наклонная. Угол между прямой и плоскостью.

Двугранный угол. Линейный угол двугранного угла. Перпендикулярность плоскостей.

Признак перпендикулярности двух плоскостей. Расстояние между точками, прямыми и

плоскостями.

Цель: дать учащимся систематические знания о перпендикулярности прямых и

плоскостей в пространстве; ввести понятие углов между прямыми и плоскостями.

О с н о в н а я ц е л ь – сформировать представления учащихся о понятиях

перпендикулярности прямых и плоскостей в пространстве, систематически изучить

Page 12: 1. Пояснительная запискаjar.edunoskol.ru/images/dokument/RAB_PROG/geometria_coo.pdf · Стандарт основного общего образования

свойства перпендикулярных прямых и плоскостей, познакомить с понятием центрального

проектирования и научить изображать пространственные фигуры на плоскости в

центральной проекции.

В данной теме обобщаются известные из планиметрии сведения о перпендикулярных

прямых. Большую помощь при иллюстрации свойств перпендикулярности и при решении

задач могут оказать модели многогранников.

В качестве дополнительного материала учащиеся знакомятся с методом изображения

пространственных фигур, основанном на центральном проектировании. Они узнают, что

центральное проектирование используется не только в геометрии, но и в живописи,

фотографии и т.д., что восприятие человеком окружающих предметов посредством зрения

осуществляется по законам центрального проектирования. Учащиеся получают

необходимые практические навыки по изображению пространственных фигур на

плоскости в центральной проекции.

4. Многогранники (22ч).

Многогранные углы. Выпуклые многогранники и их свойства. Правильные

многогранники.

Цель: сформировать у учащихся представление об основных видах многогранников и их

свойствах; рассмотреть правильные многогранники.

О с н о в н а я ц е л ь – познакомить учащихся с понятиями многогранного угла и

выпуклого многогранника, рассмотреть теорему Эйлера и ее приложения к решению

задач, сформировать представления о правильных, полуправильных и звездчатых

многогранниках, показать проявления многогранников в природе в виде кристаллов.

Среди пространственных фигур особое значение имеют выпуклые фигуры и, в частности,

выпуклые многогранники. Теорема Эйлера о числе вершин, ребер и граней выпуклого

многогранника играет важную роль в различных областях математики и ее приложениях.

При изучении правильных, полуправильных и звездчатых многогранников следует

использовать модели этих многогранников, изготовление которых описано в учебнике, а

также графические компьютерные средства.

6.Повторение (7ч).

Цель: повторить и обобщить материал, изученный в 10 классе.

11 класс (2ч в неделю, всего 68 ч)

1. Векторы в пространстве-6 часов.

Векторы в пространстве. Коллинеарные и компланарные векторы. Параллельный перенос.

Параллельное проектирование и его свойства. Параллельные проекции плоских фигур.

Изображение пространственных фигур на плоскости. Сечения многогранников.

Исторические сведения.

Цель: сформировать у учащихся понятие вектора в пространстве; рассмотреть

основные операции над векторами.

2/. Координаты точки и координаты векторов пространстве. Движения (15 ч).

Прямоугольная система координат в пространстве. Расстояние между точками в

пространстве. Векторы в пространстве. Длина вектора. Равенство векторов. Сложение

векторов. Умножение вектора на число. Координаты вектора. Скалярное произведение

векторов.

Цель: введение понятие прямоугольной системы координат в пространстве; знакомство

с координатно-векторным методом решения задач.

Цели: сформировать у учащихся умения применять координатный и векторный методы к

решению задач на нахождение длин отрезков и углов между прямыми и векторами в

пространстве. В ходе изучения темы целесообразно использовать аналогию между

рассматриваемыми понятиями на плоскости и в пространстве. Это поможет учащимся

Page 13: 1. Пояснительная запискаjar.edunoskol.ru/images/dokument/RAB_PROG/geometria_coo.pdf · Стандарт основного общего образования

более глубоко и осознанно усвоить изучаемый материал, уяснить содержание и место

векторного и координатного методов в курсе геометрии

О с н о в н а я ц е л ь – обобщить и систематизировать представления учащихся о

декартовых координатах и векторах, познакомить с полярными и сферическими

координатами.

Изучение координат и векторов в пространстве, с одной стороны, во многом повторяет

изучение соответствующих тем планиметрии, а с другой стороны, дает алгебраический

метод решения стереометрических задач.

2.Цилиндр, конус, шар (16 ч)

Основные элементы сферы и шара. Взаимное расположение сферы и плоскости.

Многогранники, вписанные в сферу. Многогранники, описанные около сферы. Цилиндр и

конус. Фигуры вращения.

Цель: выработка у учащихся систематических сведений об основных видах тел

вращения.

Цели: дать учащимся систематические сведения об основных видах тел вращения.

Изучение круглых тел (цилиндра, конуса, шара) завершает изучение системы основных

пространственных геометрических тел. В ходе знакомства с теоретическим материалом

темы значительно развиваются пространственные представления учащихся: круглые тела

рассматривать на примере конкретных геометрических тел, изучать взаимное

расположение круглых тел и плоскостей (касательные и секущие плоскости), ознакомить с

понятиями описанных и вписанных призм и пирамид. Решать большое количество задач,

что позволяет продолжить работу по формированию логических и графических умений.

О с н о в н а я ц е л ь – сформировать представления учащихся о круглых телах, изучить

случаи их взаимного расположения, научить изображать вписанные и описанные фигуры.

В данной теме обобщаются сведения из планиметрии об окружности и круге, о взаимном

расположении прямой и окружности, о вписанных и описанных окружностях. Здесь

учащиеся знакомятся с основными фигурами вращения, выясняют их свойства, учатся их

изображать и решать задачи на фигуры вращения. Формированию более глубоких

представлений учащихся могут служить задачи на комбинации многогранников и фигур

вращения.

3. Объем и площадь поверхности (21 ч).

Понятие объема и его свойства. Объем цилиндра, прямоугольного параллелепипеда и

призмы. Принцип Кавальери. Объем пирамиды. Объем конуса и усеченного конуса.

Объем шара и его частей. Площадь поверхности многогранника, цилиндра, конуса,

усеченного конуса. Площадь поверхности шара и его частей.

Цель: систематизация изучения многогранников и тел вращения в ходе решения задач на

вычисление их объемов.

Цели: продолжить систематическое изучение многогранников и тел вращения в ходе

решения задач на вычисление их объемов.

Понятие объема вводить по аналогии с понятием площади плоской фигуры и

формулировать основные свойства объемов.

Существование и единственность объема тела в школьном курсе математики приходится

принимать без доказательства,

так как вопрос об объемах принадлежит, по существу, к трудным разделам высшей

математики. Поэтому нужные результаты устанавливать, руководствуясь больше

наглядными соображениями. Учебный материал главы в основном должен усвоиться в

процессе решения задач.

О с н о в н а я ц е л ь – сформировать представления учащихся о понятиях объема и

площади поверхности, вывести формулы объемов и площадей поверхностей основных

пространственных фигур, научить решать задачи на нахождение объемов и площадей

поверхностей.

Page 14: 1. Пояснительная запискаjar.edunoskol.ru/images/dokument/RAB_PROG/geometria_coo.pdf · Стандарт основного общего образования

Изучение объемов обобщает и систематизирует материал планиметрии о площадях

плоских фигур. При выводе формул объемов используется принцип Кавальери. Это

позволяет чисто геометрическими методами, без использования интеграла или

предельного перехода, найти объемы основных пространственных фигур, включая объем

шара и его частей.

Практическая направленность этой темы определяется большим количеством

разнообразных задач на вычисление объемов и площадей поверхностей.

Повторение (10 ч.)

Цель: повторение и систематизация материала 11 класса.

Цели: повторить и обобщить знания и умения, учащихся через решение задач по

следующим темам: метод координат в пространстве; многогранники; тела вращения;

объѐмы многогранников и тел вращения

5. Формы и средства контроля

Контрольные работы

Тестирование

Самостоятельные работы

Проверочные работы

Математические диктанты

Входной контроль

Промежуточный контроль

Итоговый контроль

Критерии и нормы оценки знаний, умений и навыков обучающихся

1. Оценка письменных контрольных работ обучающихся по математике.

Ответ оценивается отметкой «5», если:

работа выполнена полностью;

в логических рассуждениях и обосновании решения нет пробелов и ошибок;

в решении нет математических ошибок (возможна одна неточность, описка,

которая не является следствием незнания или непонимания учебного материала). Отметка «4» ставится в следующих случаях:

работа выполнена полностью, но обоснования шагов решения недостаточны

(если умение обосновывать рассуждения не являлось специальным объектом проверки);

допущены одна ошибка или есть два – три недочѐта в выкладках, рисунках,

чертежах или графиках (если эти виды работ не являлись специальным объектом

проверки).

Отметка «3» ставится, если:

допущено более одной ошибки или более двух – трех недочетов в выкладках,

чертежах или графиках, но обучающийся обладает обязательными умениями по

проверяемой теме.

Отметка «2» ставится, если:

допущены существенные ошибки, показавшие, что обучающийся не обладает

обязательными умениями по данной теме в полной мере. Отметка «1» ставится, если:

Page 15: 1. Пояснительная запискаjar.edunoskol.ru/images/dokument/RAB_PROG/geometria_coo.pdf · Стандарт основного общего образования

работа показала полное отсутствие у обучающегося обязательных знаний и

умений по проверяемой теме или значительная часть работы выполнена не

самостоятельно.

Учитель может повысить отметку за оригинальный ответ на вопрос или оригинальное

решение задачи, которые свидетельствуют о высоком математическом развитии

обучающегося; за решение более сложной задачи или ответ на более сложный вопрос,

предложенные обучающемуся дополнительно после выполнения им каких-либо других

заданий.

2. Оценка устных ответов обучающихся по математике

Ответ оценивается отметкой «5», если ученик:

полно раскрыл содержание материала в объеме, предусмотренном программой

и учебником;

изложил материал грамотным языком, точно используя математическую

терминологию и символику, в определенной логической последовательности;

правильно выполнил рисунки, чертежи, графики, сопутствующие ответу;

показал умение иллюстрировать теорию конкретными примерами, применять

ее в новой ситуации при выполнении практического задания;

продемонстрировал знание теории ранее изученных сопутствующих тем,

сформированность и устойчивость используемых при ответе умений и навыков;

отвечал самостоятельно, без наводящих вопросов учителя;

возможны одна – две неточности при освещение второстепенных вопросов

или в выкладках, которые ученик легко исправил после замечания учителя.

Ответ оценивается отметкой «4», если удовлетворяет в основном требованиям на

оценку «5», но при этом имеет один из недостатков:

в изложении допущены небольшие пробелы, не исказившее математическое

содержание ответа;

допущены один – два недочета при освещении основного содержания ответа,

исправленные после замечания учителя;

допущены ошибка или более двух недочетов при освещении второстепенных

вопросов или в выкладках, легко исправленные после замечания учителя.

Отметка «3» ставится в следующих случаях:

неполно раскрыто содержание материала (содержание изложено

фрагментарно, не всегда последовательно), но показано общее понимание вопроса и

продемонстрированы умения, достаточные для усвоения программного материала

(определены «Требованиями к математической подготовке обучающихся» в настоящей

программе по математике);

имелись затруднения или допущены ошибки в определении математической

терминологии, чертежах, выкладках, исправленные после нескольких наводящих

вопросов учителя;

ученик не справился с применением теории в новой ситуации при выполнении

практического задания, но выполнил задания обязательного уровня сложности по данной

теме;

при достаточном знании теоретического материала выявлена недостаточная

сформированность основных умений и навыков.

Page 16: 1. Пояснительная запискаjar.edunoskol.ru/images/dokument/RAB_PROG/geometria_coo.pdf · Стандарт основного общего образования

Отметка «2» ставится в следующих случаях:

не раскрыто основное содержание учебного материала;

обнаружено незнание учеником большей или наиболее важной части учебного

материала;

допущены ошибки в определении понятий, при использовании математической

терминологии, в рисунках, чертежах или графиках, в выкладках, которые не исправлены

после нескольких наводящих вопросов учителя.

Отметка «1» ставится, если:

ученик обнаружил полное незнание и непонимание изучаемого учебного

материала или не смог ответить ни на один из поставленных вопросов по изученному

материалу.

3. Общая классификация ошибок.

При оценке знаний, умений и навыков обучающихся следует учитывать все ошибки

(грубые и негрубые) и недочѐты.

3.1. Грубыми считаются ошибки:

- незнание определения основных понятий, законов, правил, основных положений

теории, незнание формул, общепринятых символов обозначений величин, единиц их

измерения;

- незнание наименований единиц измерения;

- неумение выделить в ответе главное;

- неумение применять знания, алгоритмы для решения задач;

- неумение делать выводы и обобщения;

- неумение читать и строить графики;

- неумение пользоваться первоисточниками, учебником и справочниками;

- потеря корня или сохранение постороннего корня;

- отбрасывание без объяснений одного из них;

- равнозначные им ошибки;

- вычислительные ошибки, если они не являются опиской;

- логические ошибки.

3.2. К негрубым ошибкам следует отнести:

- неточность формулировок, определений, понятий, теорий, вызванная неполнотой

охвата основных признаков определяемого понятия или заменой одного - двух из этих

признаков второстепенными;

- неточность графика;

- нерациональный метод решения задачи или недостаточно продуманный план

ответа (нарушение логики, подмена отдельных основных вопросов второстепенными);

- нерациональные методы работы со справочной и другой литературой;

- неумение решать задачи, выполнять задания в общем виде.

3.3. Недочетами являются:

- нерациональные приемы вычислений и преобразований;

- небрежное выполнение записей, чертежей, схем, графиков.

Page 17: 1. Пояснительная запискаjar.edunoskol.ru/images/dokument/RAB_PROG/geometria_coo.pdf · Стандарт основного общего образования

КОНТРОЛЬНЫЕ РАБОТЫ

Геометрия 10 класс

Контрольная работа № 1

по теме «Параллельность прямых и плоскостей»

Вариант 1

1. Основание АD трапеции АВСD лежит в плоскости . Через точки В и С проведены

параллельные прямые, пересекающие плоскость в точках Е и F соответственно.

а) Каково взаимное расположение прямых ЕF и АВ ?

б) Чему равен угол между прямыми ЕF и АВ, если АВС = 150о ? Ответ

обоснуйте.

2. Дан пространственный четырѐхугольник АВСD, в котором диагонали АС и ВD равны.

Середины сторон этого четырѐхугольника соединены последовательно отрезками.

а) Выполните рисунок к задаче.

б)* Докажите, что полученный четырѐхугольник – ромб.

Геометрия 10 класс

Контрольная работа № 1

по теме «Параллельность прямых и плоскостей»

Вариант 2

1. Треугольники АВС и АDС лежат в разных плоскостях и имеют общую сторону АС.

Точка Р – середина стороны АD, точка К – середина DC.

а) Каково взаимное расположение прямых РК и АВ ?

б) Чему равен угол между прямыми РК и АВ, если АВС = 40о и ВСА = 80

о ?

Ответ обоснуйте.

2. Дан пространственный четырѐхугольник АВСD, M и N – середины сторон АВ и ВС

соответственно, Е СD, К DА, DЕ : ЕС = 1 : 2, DК : КА = 1 : 2.

а) Выполните рисунок к задаче.

б)* Докажите, что четырѐхугольник MNEK – трапеция.

Геометрия 10 класс

Контрольная работа № 2

по теме «Параллельность прямых и плоскостей»

Вариант 1

1. Прямые a и b лежат в параллельных плоскостях и . Могут ли эти прямые быть:

а) параллельными;

б) скрещивающимися?

Сделайте рисунок для каждого возможного случая.

Page 18: 1. Пояснительная запискаjar.edunoskol.ru/images/dokument/RAB_PROG/geometria_coo.pdf · Стандарт основного общего образования

2. Через точку О, лежащую между параллельными плоскостями и , проведены

прямые l и m . Прямая l пересекает плоскости и в точках А1 и А2

соответственно, прямая m - в точках В1 и В2. Найдите длину отрезка А2В2, если

А1В1 = 12 см, В1О : ОВ2 = 3 : 4.

3*. Изобразите параллелепипед АВСDА1В1С1D1 и постройте его сечение плоскостью,

проходящей через точки M, N и К, являющиеся серединами рѐбер АВ, ВС и DD1.

Геометрия 10 класс

Контрольная работа № 2

по теме «Параллельность прямых и плоскостей»

Вариант 2

1. Прямые a и b лежат в пересекающихся плоскостях и . Могут ли эти прямые

быть:

а) параллельными;

б) скрещивающимися?

Сделайте рисунок для каждого возможного случая.

2. Через точку О, не лежащую между параллельными плоскостями и , проведены

прямые l и m . Прямая l пересекает плоскости и в точках А1 и А2

соответственно, прямая m - в точках В1 и В2. Найдите длину отрезка А1В1, если

А2В2 = 15 см, О В1 : ОВ2 = 3 : 5.

3*. Изобразите тетраэдр DABC и постройте его сечение плоскостью, проходящей через

точки M и N, являющиеся серединами рѐбер DC и ВС, и точку К, такую, что К DA,

AK : KD = 1 : 3.

Геометрия 10 класс

Контрольная работа № 3

по теме «Перпендикулярность прямых и плоскостей»

Вариант 1

1. Диагональ куба равна 6 см. Найдите:

а) ребро куба;

б) косинус угла между диагональю куба и плоскостью одной из его граней.

2. Сторона АВ ромба АВСD равна а , один из углов ромба равен 60о. Через сторону АВ

проведена плоскость на расстоянии 2

а от точки D.

а) Найдите расстояние от точки С до плоскости .

б) Покажите на рисунке линейный угол двугранного угла DАВМ, М .

в)* Найдите синус угла между плоскостью ромба и плоскостью .

Геометрия 10 класс

Page 19: 1. Пояснительная запискаjar.edunoskol.ru/images/dokument/RAB_PROG/geometria_coo.pdf · Стандарт основного общего образования

Контрольная работа № 3

по теме «Перпендикулярность прямых и плоскостей»

Вариант 2

1. Основанием прямоугольного параллелепипеда служит квадрат, диагональ

параллелепипеда равна 62 см, а его измерения относятся как 1 : 1 : 2. Найдите:

а) измерения параллелепипеда;

б) косинус угла между диагональю параллелепипеда и плоскостью его основания.

2. Сторона квадрата АВСD равна а . Через сторону АD проведена плоскость на

расстоянии 2

а от точки В.

а) Найдите расстояние от точки С до плоскости .

б) Покажите на рисунке линейный угол двугранного угла ВАDМ, М .

в)* Найдите синус угла между плоскостью квадрата и плоскостью .

Геометрия 10 класс

Контрольная работа № 4

по теме «Многогранники»

Вариант 1

1. Основанием пирамиды DАВС является правильный треугольник АВС, сторона

которого равна а . Ребро DА перпендикулярно к плоскости АВС, а плоскость DВС

составляет с плоскостью АВС угол 30о. Найдите площадь боковой поверхности

пирамиды.

2. Основанием прямого параллелепипеда АВСDА1В1С1D1 является ромб АВСD, сторона

которого равна а и угол равен 60о. Плоскость АD1С1 составляет с плоскостью

основания угол 60о. Найдите:

а) высоту ромба;

б) высоту параллелепипеда;

в) площадь боковой поверхности параллелепипеда;

г)* площадь поверхности параллелепипеда.

Геометрия 10 класс

Контрольная работа № 4

по теме «Многогранники»

Вариант 2

1. Основанием пирамиды МАВСD является квадрат АВСD, ребро МD перпендикулярно к

плоскости основания, AD = DM = а . Найдите площадь поверхности пирамиды.

Page 20: 1. Пояснительная запискаjar.edunoskol.ru/images/dokument/RAB_PROG/geometria_coo.pdf · Стандарт основного общего образования

2. Основанием прямого параллелепипеда АВСDА1В1С1D1 является параллелограмм

АВСD, стороны которого равны 2а и а2 , острый угол равен 45о. Высота

параллелепипеда равна меньшей высоте параллелограмма. Найдите:

а) меньшую высоту параллелограмма;

б) угол между плоскостью АВС1 и плоскостью основания;

в) площадь боковой поверхности параллелепипеда;

г)* площадь поверхности параллелепипеда.

ЗАЧЁТЫ

Геометрия 10 класс

Зачет № 1

по теме «Параллельность прямых и плоскостей»

Вариант 1

1. Дана прямая a и точка К, которая не лежит на этой прямой. Через точку К

проведены прямые m и l, пересекающие прямую a. Докажите, что прямые a, m и l

лежат в одной плоскости.

2. Можно ли провести через середину стороны треугольника прямую, которая не

имеет общих точек с другой его стороной? Поясните ответ.

3. Основание AD трапеции ABCD лежит в плоскости . Вершина C не лежит в этой

плоскости. Через середины боковых сторон трапеции проведена прямая m.

Докажите, что прямая m параллельна плоскости .

4. Дан треугольник MPK. Плоскость, параллельная прямой MK, пересекает сторону

MP в точке M1, а сторону PK – в точке K1. Вычислите длину отрезка M1K1, если

MK = 27 см, PK1 : K1K = 5 : 4.

5. Точка О не лежит в плоскости параллелограмма ABCD. Как расположены прямые

AB и p, проходящая через середины отрезков OC и OD? Найдите угол между

прямыми p и BC, если BAD = 130о.

6. Через точку К, лежащую между параллельными плоскостями и , проведены

прямые a и b. Первая прямая пересекает плоскости и β в точках А1 и В1

соответственно, вторая – в точках А2 и В2. Вычислите длину отрезка КВ2, если

А1А2 : В1В2 = 3 : 5, А2В2 = 16 см.

7. Дан параллелепипед ABCDA1B1C1D1. Постройте сечение этого параллелепипеда

плоскостью, проходящей через середину ребра AB и параллельной плоскости

АСС1.

8. Верно ли утверждение, что прямая, лежащая в одной из параллельных плоскостей,

параллельна другой плоскости? Ответ обоснуйте.

Page 21: 1. Пояснительная запискаjar.edunoskol.ru/images/dokument/RAB_PROG/geometria_coo.pdf · Стандарт основного общего образования

Геометрия 10 класс

Зачет № 1

по теме «Параллельность прямых и плоскостей»

Вариант 2

1. Прямая m пересекает лучи АВ, АС и AD в точках К, Р и Т. Докажите, что точки А,

К, Р и Т лежат в одной плоскости.

2. Можно ли провести через точку пересечения диагоналей прямоугольника прямую,

которая не имеет с его сторонами общих точек? Поясните ответ.

3. Вершины В и С треугольника АВС лежат в плоскости β. Вершина А ей не

принадлежит. Докажите, что прямая, проходящая через середины отрезков АВ и

АС, параллельна плоскости β.

4. Дан треугольник АВС. Плоскость, параллельная прямой АС, пересекает сторону

АВ в точке А1, а сторону ВС – в точке С1. Вычислите длину отрезка ВС1, если

СС1 = 20 см, А1С1 : АС = 3 : 7.

5. Точка О не принадлежит плоскости равнобедренной трапеции КМРТ (КТ || МР).

Как расположены прямые, одна из которых содержит среднюю линию трапеции, а

другая – середины отрезков ОМ и ОР? Найдите угол между прямой МК и прямой,

содержащей середины отрезков ОМ и ОР, если МРТ = 110о.

6. Луч КМ пересекает параллельные плоскости и в точках М1 и М2, а луч КР –

в точках Р1 и Р2 соответственно. Вычислите длину отрезка М1М2, если КМ1 = 8 см,

М1Р1 : М2Р2 = 4 : 9.

7. Дан тетраэдр АВСD. Точка М – середина ребра DC, точка К – середина ребра AD.

Постройте сечение тетраэдра плоскостью, содержащей точку К и параллельной

плоскости АМВ.

8. Прямые а и b расположены соответственно в параллельных плоскостях и .

Верно ли, что эти прямые не имеют общих точек? (Ответ обоснуйте).

Геометрия 10 класс

Зачет № 1

по теме «Параллельность прямых и плоскостей»

Вариант 3

1. Точки М, N и Р лежат на прямой а. Точка А не лежит на этой прямой. Докажите,

что точки А, М, N и Р расположены в одной плоскости.

2. Можно ли провести через середину медианы треугольника прямую, которая не

имеет общих точек с его сторонами? Поясните ответ.

Page 22: 1. Пояснительная запискаjar.edunoskol.ru/images/dokument/RAB_PROG/geometria_coo.pdf · Стандарт основного общего образования

3. Точка М не принадлежит плоскости прямоугольника АВСD. Прямая а проходит

через точку М и параллельна прямой АС. Докажите, что прямая, проходящая через

середины отрезков МА и МС, параллельна плоскости прямоугольника.

4. Дан треугольник СКР. Плоскость, параллельная прямой РК, пересекает сторону СР

в точке Е, а сторону КС – в точке F. Вычислите длину отрезка РК, если ЕF = 14 дм,

СЕ : ЕР = 2 : 5.

5. Точка А не лежит в плоскости ромба ВСDЕ. Как расположены прямая ВD и прямая

m, которая проходит через середины отрезков АВ и АС? Найдите угол между

прямыми т и ВD, если СDЕ = 120о.

6. Через точку D, лежащую между параллельными плоскостями и β, проведены

прямые т и k. Прямая т пересекает плоскости и β в точках М1 и М2

соответственно, а прямая k – в точках К1 и К2 соответственно. Вычислите длину

отрезка DМ2, если М1М2 = 20 дм, М1К1 : М2К2 = 3 : 7.

7. Дан параллелепипед MNPQM1N1P1Q1. Постройте сечение параллелепипеда

плоскостью, проходящей через середину ребра MN и параллельной плоскости

QNN1.

8. В одной из параллельных плоскостей проведена прямая. Верно ли, что она

параллельна второй плоскости? (Ответ обоснуйте).

Геометрия 10 класс

Зачет № 1

по теме «Параллельность прямых и плоскостей»

Вариант 4

1. 1. Прямые k и l пересекаются в точке О. Прямая а пересекает их в точках М и Р, а

прямая b – в точках С и D. Докажите, что прямые a и b лежат в одной плоскости.

2. Можно ли провести через середину диагонали квадрата прямую, которая не имеет

общих точек с его сторонами? Поясните ответ.

3. Отрезок МР расположен в плоскости . Точка А не лежит в ней. Докажите, что

прямая, проходящая через середины отрезков АМ и АР, параллельна плоскости .

4. Дан треугольник ЕFТ. Плоскость, параллельная прямой FT, пересекает сторону EF

в точке D, а сторону ЕТ – в точке С. Вычислите длину отрезка СD, если FT = 24 дм,

DE : EF = 1 : 3.

5. Точка М не лежит в плоскости квадрата ABCD. Как расположены прямая АС и

прямая, проходящая через середины отрезков МА и МВ? Найдите угол между

этими прямыми.

Page 23: 1. Пояснительная запискаjar.edunoskol.ru/images/dokument/RAB_PROG/geometria_coo.pdf · Стандарт основного общего образования

6. Луч АВ пересекает параллельные плоскости и β в точках В1 и В2 соответственно,

а луч АС пересекает их в точках С1 и С2 соответственно. Вычислите длину

отрезка С1С2, если АС1 = 9 дм, В1С1 : В2С2 = 3 : 5.

7. Дан тетраэдр МКРТ. Точка А – середина ребра МР, точка В – середина ребра РТ.

Постройте сечение тетраэдра плоскостью, содержащей точки А, В и параллельной

плоскости МКТ.

8. Плоскости и β параллельны. Прямая т лежит в плоскости , а прямая k – в

плоскости β. Верно ли, что прямые т и k не пересекаются? (Ответ обоснуйте).

Геометрия 10 класс

Зачет № 2

по теме «Перпендикулярность прямых и плоскостей»

Вариант 1

1. Какое из следующих утверждений неверно?

А) Если одна из двух параллельных прямых перпендикулярна третьей прямой, то и

другая прямая перпендикулярна к этой прямой.

Б) Прямая называется параллельной плоскости, если она перпендикулярна к любой

прямой, лежащей в этой плоскости.

В) Две прямые, перпендикулярные к плоскости, параллельны.

Г) Если одна из двух параллельных прямых перпендикулярна к плоскости, то и

другая прямая перпендикулярна к этой плоскости.

Д) Через любую точку пространства проходит прямая, перпендикулярная к данной

плоскости, и притом только одна.

2. Две скрещивающиеся прямые взаимно перпендикулярны. Чему равен угол между

ними?

А) 90о. Б) 0

о. В) 180

о. Г) 45

о. Д) Определить нельзя.

3. Через вершину квадрата АВСD проведена прямая АМ, перпендикулярная его

плоскости. Какое из следующих утверждений неверно?

А) МАВD. Б) МDСD. В) МВВС. Г) МСВС. Д) МА АС.

4. Прямая т перпендикулярна к прямым а и b, лежащим в плоскости , но т не

перпендикулярна к плоскости .Выясните взаимное расположение прямых а и b.

А) Параллельны. Б) Пересекаются. В) Скрещиваются.

Г) Совпадают. Д) Определить нельзя.

5. Прямая перпендикулярна к двум плоскостям, тогда плоскости:

А) пересекаются; Б) параллельны; В) определить нельзя;

Г) скрещиваются; Д) совпадают.

6. В тетраэдре DABC АDАС, АDАВ, DСВС. Тогда прямая ВС и плоскость ADC:

А) параллельны;

Б) прямая лежит в плоскости;

В) прямая пересекает плоскость, но не перпендикулярна к плоскости;

Г) прямая перпендикулярна к плоскости, но не пересекает плоскость.

Page 24: 1. Пояснительная запискаjar.edunoskol.ru/images/dokument/RAB_PROG/geometria_coo.pdf · Стандарт основного общего образования

7. Какое из следующих утверждений неверно?

А) Перпендикуляр и наклонная, выходящие из одной точки, имеют разную длину.

Б) Расстоянием от точки до плоскости называется длина перпендикуляра,

проведенного из данной точки к данной плоскости.

В) равные наклонные, проведенные к плоскости из одной точки, имеют разные

проекции.

Г) Проекцией точки на плоскость является точка.

Д) Углом между прямой и плоскостью, пересекающей эту прямую и

неперпендикулярной к ней, называется угол между прямой и ее проекцией на эту

плоскость.

8. Расстояния от точки М до вершин прямоугольного треугольника АВС ( С = 90о)

равны. Какое из следующих утверждений верно?

А) Плоскости МАВ и АВС перпендикулярны.

Б) Плоскости МВС и АВС перпендикулярны.

В) Плоскости МАС и АВС перпендикулярны.

Г) Плоскости МАС и МВС перпендикулярны.

Д) Условия в пунктах А-Г неверны.

9. Плоскости и пересекаются по прямой с. Плоскость перпендикулярна плоскости

, но не перпендикулярна плоскости . Выясните взаимное расположение прямой с и

плоскости .

А) .с Б) с . В) .с Г) Определить нельзя.

Д) С уверенностью можно сказать только то, что прямая с не перпендикулярна

плоскости .

10. При пересечении двух плоскостей образовались двугранные углы, один из которых в 2

раза больше другого. Найдите градусную меру угла между этими плоскостями.

А) 30о. Б) 60

о. В) 90

о. Г) 120

о. Д) 150

о.

11. Равнобедренные треугольники АВС и ВDС, каждый из которых имеет основание ВС,

не лежат в одной плоскости. Их высоты, проведенные к основанию, равны 5 см, и

расстояние между точками А и D также равно 5 см. Найдите градусную меру двугранного

угла АВСD.

А) 60о. Б) 120

о. В) 30

о. Г) 45

о. Д) 90

о.

12. Какое из следующих утверждений верно?

А) Двугранным углом называется фигура, образованная прямой а и двумя

полуплоскостями с общей границей а.

Б) Двугранный угол имеет бесконечное множество различных линейных углов.

В) Градусной мерой двугранного угла называется градусная мера его линейного

угла.

Г) Угол между пересекающимися плоскостями может быть тупым.

Д) Если одна из двух плоскостей проходит через прямую, пересекающую другую

плоскость, то такие плоскости перпендикулярны.

Геометрия 10 класс

Page 25: 1. Пояснительная запискаjar.edunoskol.ru/images/dokument/RAB_PROG/geometria_coo.pdf · Стандарт основного общего образования

Зачет № 2

по теме «Перпендикулярность прямых и плоскостей»

Вариант 2

1. Если угол между двумя прямыми равен 90о, то эти прямые:

А) пересекаются; Б) параллельны; В) скрещиваются;

Г) перпендикулярны; Д) совпадают.

2. Какое из следующих утверждений неверно?

А) Если прямая перпендикулярна к двум прямым, лежащим в плоскости, то она

перпендикулярна к этой плоскости.

Б) Если прямая перпендикулярна к плоскости, то она ее пересекает.

В) Если две плоскости перпендикулярны к прямой, то они параллельны.

Г) Если две прямые перпендикулярны к плоскости, то они параллельны.

Д) Если одна из двух параллельных прямых перпендикулярна к плоскости, то и

другая прямая перпендикулярна к этой плоскости.

3. Если одна из двух скрещивающихся прямых перпендикулярна к плоскости, то будет ли

перпендикулярна к этой плоскости вторая прямая?

А) Да. Б) Да, но при определенных условиях.

В) Определить нельзя. Г) Нет. Д) Другой ответ.

4. Прямая а перпендикулярна к прямым с и b, лежащим в плоскости , прямая а

перпендикулярна к плоскости . Каково взаимное расположение прямых с и b?

А) параллельны. Б) Пересекаются. В) параллельны или пересекаются.

Г) Совпадают. Д) Определить нельзя.

5. Одна из двух параллельных плоскостей перпендикулярна прямой. Тогда:

А) другая плоскость параллельна прямой;

Б) прямая лежит в другой плоскости;

В) другая плоскость перпендикулярна прямой;

Г) прямая не пересекает другую плоскость;

Д) выполняются все случаи, указанные в пунктах А-Г.

6. Точка Е не принадлежит плоскости прямоугольника АВСD. ВЕ АВ, ВЕ ВС. Тогда

прямая СD и плоскость ВСЕ:

А) параллельны; Б) перпендикулярны; В) скрещиваются;

Г) прямая лежит в плоскости; Д) перпендикулярны, но не пересекаются.

7. Какое из следующих утверждений неверно?

А) Перпендикуляр и наклонная, выходящие из одной точки, имеют равные длины.

Б) Проекцией прямой на плоскость является точка или прямая.

В) Наклонные разной длины, проведенные к плоскости из одной точки, имеют

проекции разных длин.

Г) Прямая, проведенная в плоскости через основание наклонной перпендикулярно

к ней, перпендикулярна к ее проекции.

Д) расстояние от произвольной точки одной из параллельных плоскостей до другой

плоскости называется расстоянием между параллельными плоскостями.

8. Расстояния от точки М до сторон прямоугольного треугольника АВС ( С = 90о)

равны. Какое из следующих утверждений верно?

Page 26: 1. Пояснительная запискаjar.edunoskol.ru/images/dokument/RAB_PROG/geometria_coo.pdf · Стандарт основного общего образования

А) Плоскости МАВ и АВС перпендикулярны.

Б) Плоскости МВС и АВС перпендикулярны.

В) Плоскости МАС и АВС перпендикулярны.

Г) Плоскости МАС и МВС перпендикулярны.

Д) Условия в пунктах А-Г неверны.

9. Угол между двумя плоскостями равен 80о. Какое из следующих утверждений неверно?

А) Плоскости пересекаются.

Б) В одной из плоскостей найдется прямая, перпендикулярная другой плоскости.

В) В одной из плоскостей все прямые не перпендикулярны другой плоскости.

Г) В одной из плоскостей найдется прямая, параллельная другой плоскости.

Д) Плоскости не перпендикулярны.

10. При пересечении двух плоскостей образовались двугранные углы, градусная мера

одного из которых на 30о больше градусной меры другого. Найдите градусную меру угла

между этими плоскостями.

А) 105о. Б) 90

о. В) 75

о. Г) 60

о. Д) 45

о.

11. Равнобедренные треугольники АВС и ВDС, каждый из которых имеет основание ВС,

не лежат в одной плоскости. Их высоты, проведенные к основанию, равны 2 см, а

расстояние между точками А и D равно 2 2 см. Найдите градусную меру двугранного

угла АВСD.

А) 60о. Б) 120

о. В) 30

о. Г) 45

о. Д) 90

о.

12. Какое из следующих утверждений верно?

А) Градусная мера двугранного угла не превосходит 90о.

Б) Двугранным углом называется плоский угол, образованный прямой а и двумя

полуплоскостями с общей границей а.

В) Если одна из двух плоскостей проходит через прямую, перпендикулярную к

другой плоскости, то такие плоскости перпендикулярны.

Г) Угол между плоскостями всегда тупой.

Д) Все линейные углы двугранного угла различны.

Геометрия 10 класс

Зачет № 2

по теме «Перпендикулярность прямых и плоскостей»

Вариант 3

1. Дан правильный треугольник АВС со стороной, равной 3. Точка О – центр

треугольника, ОМ – перпендикуляр к его плоскости, ОМ = 1. Найдите расстояния от

точки М до вершин треугольника.

А) 3 . Б) Определить нельзя. В) 3. Г) 1. Д) 2.

2. Отрезок АВ, равный 5 см, не имеет общих точек с плоскостью . Прямые АС и ВD,

перпендикулярные к этой плоскости, пересекают ее в точках С и D соответственно.

Найдите ВD, если СD = 3 см, АС = 17 см, ВD < AC.

А) Определить нельзя. Б) 12 см. В) 13 см. Г) 17 - 34 см. Д) 1 см.

Page 27: 1. Пояснительная запискаjar.edunoskol.ru/images/dokument/RAB_PROG/geometria_coo.pdf · Стандарт основного общего образования

3. Расстояние от некоторой точки до плоскости квадрата равно 4 см, а до каждой из его

вершин – 6 см. Найдите диагональ квадрата.

А) 2 5 см. Б) 5 см. В) 5 2 см. Г) 2 10 см. Д) 4 5 см.

4. Отрезок АВ пересекает некоторую плоскость в точке О. Прямые АD и ВС,

перпендикулярные к этой плоскости, пересекают ее в точках D и С соответственно.

Найдите длину АВ, если АD = 6 см, ВС = 2 см, ОС = 1,5 см.

А) 8 см. Б) Определить нельзя. В) 14 см. Г) 10 см. Д) 12 см.

5. Из точки М к плоскости проведены две наклонные, длины которых 18 см и 2 109

см. Их проекции на эту плоскость относятся как 3 : 4. Найдите расстояние от точки М до

плоскости .

А) 6 5 см. Б) 30 см. В) 6 см. Г) 3 14 см. Д) 2 78 см.

6. Расстояние от точки М до каждой из вершин правильного треугольника АВС равно 4

см. Найдите расстояние от точки М до плоскости АВС, если АВ = 6 см.

А) 4 см. Б) 16 – 2 3 см. В) 8 см. Г) 6 см. Д) 2 см.

7. Через точку А, удаленную от плоскости на 4 см, проходит прямая, пересекающая

плоскость в точке В. Найдите угол между прямой АВ и плоскостью , если длина

отрезка АВ равна 6 см.

А) arccos3

2. Б)

3

2arcsiп . В)

2

3arcsiп . Г)

3

2arctg . Д) arcctg

3

2.

8. Из точки к плоскости проведены две равные наклонные. Величина угла между этими

наклонными равна 60о. Величина угла между их проекциями равна 90

о. Найдите угол

между каждой наклонной и ее проекцией.

А) 90о. Б) 60

о. В) 30

о. Г) 45

о. Д) Определить нельзя.

9. Прямая СD перпендикулярна к плоскости остроугольного треугольника АВС, у

которого СК – высота. Найдите расстояние от точки А до плоскости СDК, если DА = 2

см, а DАК = 45о.

А) 2 см. Б) 2 см. В) 3 см. Г) 1 см. Д) 5 см.

10. Точка А находится на расстоянии 3 см и 5 см от двух перпендикулярных плоскостей.

Найдите расстояние от точки А до прямой пересечения этих плоскостей.

А) 34 см. Б) 4 см. В) 6 см. Г) 2 7 см. Д) 14 см.

11. Отрезок АМ является перпендикуляром к плоскости прямоугольника АВСD. Угол

между прямой МС и этой плоскостью равен 30о. АD = 2 , СD = 2. Найдите величину

двугранного угла МСDА.

А) Определить нельзя. Б) 45о. В) 30

о. Г) 60

о. Д) 90

о.

12. Найдите двугранный угол АВСD тетраэдра АВСD, если углы DАВ, DАС и АСВ

прямые, АС = ВС = 5, АD = 5 2 .

А) arcсos 5 2 . Б) arcsin5 2 . В) arcctg 2 . Г) arctg 2 .

Д) Определить нельзя.

Page 28: 1. Пояснительная запискаjar.edunoskol.ru/images/dokument/RAB_PROG/geometria_coo.pdf · Стандарт основного общего образования

Геометрия 10 класс

Зачет № 2

по теме «Перпендикулярность прямых и плоскостей»

Вариант 4

1. АВСD – квадрат со стороной, равной 2 , О – точка пересечения его диагоналей, ОЕ –

перпендикуляр к плоскости АВС, ОЕ = 3 . Найдите расстояние от точки Е до вершин

квадрата.

А) Определить нельзя. Б) 2 . В) 3 . Г) 1. Д) 2.

2. Отрезок МН не имеет общих точек с плоскостью . Прямые МК и НТ,

перпендикулярные к этой плоскости, пересекают ее в точках К и Т соответственно.

Найдите МН, если КТ = 5 см, МК = 4 см, НТ = 6 см.

А) 29 см. Б) 7 см. В) 3 3 см. Г) 3 см. Д) Определить нельзя.

3. Расстояние от некоторой точки до плоскости квадрата равно 4 см., а до каждой из его

сторон – 6 см. Найдите диагональ квадрата.

А) 2 10 см. Б) 5 2 см. В) 5 10 см. Г) 10 2 см. Д) 4 10 см.

4. Отрезок МН пересекает некоторую плоскость в точке К. Через концы отрезка

проведены прямые НР и МЕ, перпендикулярные к плоскости и пересекающие ее в точках

Р и Е соответственно. Найдите длину отрезка РЕ, если НР = 4 см, НК = 5 см, МЕ = 12 см.

А) Определить нельзя. Б) 8 см. В) 10 см. Г) 12 см. Д) 14 см.

5. Из точки М к плоскости проведены две наклонные, длины которых 18 см и 2 53 см.

Их проекции на эту плоскость относятся как 4 : 3. Найдите расстояние от точки М до

плоскости .

А) 34 см. Б) 2 17 см. В) 2 см. Г) 2 77 см. Д) 10 2 см.

6. Расстояние от точки К до каждой из вершин квадрата АВСD равно 4 см. Найдите

расстояние от этой точки до плоскости АВС, если АВ = 2 см.

А) 4 – 2 см. Б) 14 см. В) 2 см. Г) 14 см. Д) 2 5 см.

7. Из точки к плоскости проведены две равные наклонные. Величина угла между этими

наклонными равна 60о. Найдите величину угла между их проекциями, если угол между

каждой наклонной и ее проекцией равен 45о.

А) 30о. Б) 45

о. В) 60

о. Г) 90

о. Д) Определить нельзя.

8. Треугольник АВС – прямоугольный (С = 90о), А = 30

о, АВ = 12. Точка М удалена

на расстояние, равное 10, от каждой вершины треугольника. Найдите угол между прямой

МС и плоскостью АВС.

А) arcsin 0,8. Б) arccos 0,8. В) arctg 0,8. Г) arcctg 0,8. Д) arcsin 0,6.

9. Прямая СD перпендикулярна к плоскости остроугольного треугольника АВС, у

которого СК – высота. Расстояние от точки А до плоскости DКС равно 2 см. Найдите

длину DА, если DАК = 45о.

А) 2 см. Б) 2 см. В) 1 см. Г) 3 см. Д) 5 см.

Page 29: 1. Пояснительная запискаjar.edunoskol.ru/images/dokument/RAB_PROG/geometria_coo.pdf · Стандарт основного общего образования

10. Точка А находится на расстоянии 1 см от одной из двух перпендикулярных

плоскостей. Найдите расстояние от точки А до второй плоскости, если расстояние до

прямой их пересечения равно 5 см.

А) 2 см. Б) 2 см. В) 1 см. Г) 3 см. Д) 4 см.

11. В треугольнике АВС угол В – прямой, ВС = 2. Проекцией этого треугольника на

некоторую плоскость является треугольник ВСD, АD = 2 . Двугранный угол АВСD

равен 45о. Найдите угол между прямой АС и плоскостью ВСD.

А) 30о. Б) 45

о. В) Определить нельзя. Г) 60

о. Д) 90

о.

12. Найдите двугранный угол DАВС тетраэдра АВСD, если ребро DС перпендикулярно к

плоскости АВС, АС = ВС = АВ = 6, ВD = 3 7 .

А) 90о

. Б) 60о. В) 45

о. Г) 120

о. Д) 30

о.

Зачет № 3 по теме «Многогранники»

Вариант 1 1. Какое из следующих утверждений неверно?

А) Параллелепипед называется прямоугольным, если его боковые ребра перпендикулярны к

основанию, а основания представляют собой прямоугольники.

Б) В прямоугольном параллелепипеде все шесть граней – произвольные параллелограммы.

В) Все двугранные углы прямоугольного параллелепипеда прямые.

Г) Куб является прямоугольным параллелепипедом.

Д) Квадрат диагонали прямоугольного параллелепипеда равен сумме квадратов трех его

измерений.

2. Измерениями прямоугольного параллелепипеда называются:

А) длины трех произвольно взятых диагоналей;

Б) длины трех равных ребер параллелепипеда;

В) длины трех ребер, имеющих общую вершину;

Г) длины диагоналей основания параллелепипеда;

Д) длины смежных сторон и диагонали параллелепипеда.

3. Найдите длину диагонали прямоугольного параллелепипеда, если его измерения равны 2 м, 3 м

и 5 м.

А) 10 м; Б) 38 м; В) 10 м; Г) 38 м; Д) 4 2 м.

4. Сколько ребер у шестиугольной призмы?

А) 18; Б) 6; В) 24; Г) 12; Д) 15.

5. Какое наименьшее число граней может иметь призма?

А) 3; Б) 4; В) 5; Г) 6; Д) 9.

6. Выберите верное утверждение:

А) у п-угольной призмы 2п граней;

Б) призма называется правильной, если ее основания – правильные многоугольники;

В) у треугольной призмы нет диагоналей;

Г) высота призмы равна ее боковому ребру;

Д) площадью боковой поверхности призмы называется сумма площадей всех ее граней.

7. Сколько ребер у шестиугольной пирамиды?

Page 30: 1. Пояснительная запискаjar.edunoskol.ru/images/dokument/RAB_PROG/geometria_coo.pdf · Стандарт основного общего образования

А) 6; Б) 12; В) 18; Г) 24; Д) 8.

8. Какое наименьшее число граней может иметь пирамида?

А) 5; Б) 12; В) 10; Г) 6; Д) 4.

9. Выберите верное утверждение:

А) Многогранник, составленный из п треугольников, называется пирамидой.

Б) Все боковые ребра усеченной пирамиды равны.

В) Пирамида называется правильной, если ее основание – правильный многоугольник.

Г) Высота боковой грани правильной пирамиды, проведенная из ее вершины, называется

апофемой.

Д) Площадью боковой поверхности усеченной пирамиды называется сумма площадей ее

граней.

10. Какое из перечисленных геометрических тел не является правильным многогранником?

А) правильный тетраэдр; Б) правильный гексаэдр; В) правильная призма;

Г) правильный додекаэдр; Д) правильный октаэдр.

11. Выберите верное утверждение:

А) Выпуклый многогранник называется правильным, если его грани – равные

многоугольники, и в каждой его вершине сходится одно и то же число ребер.

Б) Не существует правильного многогранника, гранями которого являются правильные

шестиугольники.

В) Правильная треугольная пирамида и правильный тетраэдр – это одно и то же.

Г) Из всех правильных многогранников только правильный тетраэдр имеет центр симметрии.

Д) Разверткой боковой поверхности куба является правильный треугольник.

12. Какое из следующих утверждений неверно?

А) Сумма двугранных углов правильного тетраэдра и правильного октаэдра равна 180о.

Б) Центры граней куба являются вершинами правильного октаэдра.

В) Правильный додекаэдр состоит из 12 правильных пятиугольников.

Г) Сумма плоских углов при каждой вершине правильного икосаэдра равна 270о.

Д) Куб и правильный гексаэдр – это одно и то же.

Зачет № 3 по теме «Многогранники» Вариант 2

1. Какое из следующих утверждений верно?

А) В прямоугольном параллелепипеде все шесть граней – произвольные параллелограммы.

Б) Все двугранные углы параллелепипеда острые.

В) Прямоугольный параллелепипед, у которого все три измерения равны, называется кубом.

Г) Квадрат диагонали прямоугольного параллелепипеда равен сумме трех его измерений.

Д) Параллелепипед называется прямоугольным, если его боковые ребра перпендикулярны к

основанию.

2. Длины трех ребер, имеющих общую вершину, называются:

А) высотами прямоугольного параллелепипеда;

Б) диагоналями прямоугольного параллелепипеда;

В) измерениями прямоугольного параллелепипеда;

Page 31: 1. Пояснительная запискаjar.edunoskol.ru/images/dokument/RAB_PROG/geometria_coo.pdf · Стандарт основного общего образования

Г) диагоналями основания прямоугольного параллелепипеда;

Д) смежными ребрами прямоугольного параллелепипеда.

3. Найдите длину диагонали прямоугольного параллелепипеда, если его измерения равны 3 см, 4

см и 5 см.

А) 5 2 см; Б) 2 3 см; В) 50 см; Г) 12 см; Д) 4 2 см.

4. Сколько граней у шестиугольной призмы?

А) 6; Б) 8; В) 10; Г) 12; Д) 16.

5. Какое наименьшее число ребер может иметь призма?

А) 9; Б) 8; В) 7; Г) 6; Д) 5.

6. Выберите верное утверждение:

А) у п-угольной призмы 2п ребер;

Б) площадью полной поверхности призмы называется сумма площадей ее боковых граней;

В) у треугольной призмы две диагонали;

Г) высота прямой призмы равна ее боковому ребру;

Д) призма называется правильной, если в основании лежит правильный многоугольник.

7. Сколько граней у шестиугольной пирамиды?

А) 6; Б) 7; В) 8; Г) 10; Д) 12.

8. Какое наименьшее число ребер может иметь пирамида?

А) 6; Б) 5; В) 4; Г) 7; Д) 8.

9. Выберите верное утверждение:

А) Высота пирамиды называется апофемой.

Б) Боковые грани усеченной пирамиды – прямоугольники;

В) Площадь боковой поверхности пирамиды равна произведению периметра основания на

высоту.

Г) Пирамида называется правильной, если ее основание – правильный многоугольник.

Д) Усеченная пирамида называется правильной, если она получена сечением правильной

пирамиды плоскостью, параллельной основанию.

10. Какое из перечисленных геометрических тел не является правильным многогранником?

А) правильный тетраэдр; Б) правильный додекаэдр; В) правильный гексаэдр;

Г) правильная пирамида; Д) правильный октаэдр.

11. Выберите верное утверждение:

А) Правильный многогранник, у которого грани являются правильными шестиугольниками,

называется правильным гексаэдром.

Б) Сумма плоских углов при вершине правильного додекаэдра равна 324о.

В) Куб имеет два центра симметрии – по одному в каждом основании.

Г) Правильный тетраэдр состоит из 8 правильных треугольников.

Д) Всего существует 6 видов правильных многогранников.

12. Какое из следующих утверждений неверно?

А) Правильный тетраэдр не имеет центра симметрии.

Б) Центры граней куба являются вершинами правильного тетраэдра.

Page 32: 1. Пояснительная запискаjar.edunoskol.ru/images/dokument/RAB_PROG/geometria_coo.pdf · Стандарт основного общего образования

В) Центры граней правильного октаэдра являются вершинами куба.

Г) Сумма плоских углов при каждой вершине куба равна 270о.

Д) Правильная треугольная пирамида не является правильным тетраэдром.

Зачет № 3 по теме «Многогранники»

Вариант 3

1. Найдите расстояние от вершины верхнего основания куба до центра нижнего основания, если

диагональ грани куба равна 2 2 см.

А) 2 + 2 см; Б) 2 см; В) 2 см; Г) 5 см; Д) 6 см.

2. Дан прямоугольный параллелепипед АВСDA1B1C1D1, у которого BD1 = d, AC = m, AB = n.

Найдите расстояние между прямой А1С1 и плоскостью АВС.

А) Определить нельзя; Б) 𝑚2 − 𝑛2; В) 𝑑2 − 𝑛2;

Г) 𝑑2 − 𝑚2; Д) 𝑑2 − 𝑚2 − 𝑛2.

3. Сумма площадей трех граней прямоугольного параллелепипеда, имеющих общую вершину,

равна 404 дм2, а его ребра пропорциональны числам 3, 7 и 8. Найдите длину диагонали параллелепипеда.

А) Определить нельзя; Б) 2 122 дм; В) 488 дм; Г) 36 дм; Д) 4 61 дм.

4. В основании прямой призмы АВСА1В1С1 лежит равнобедренный прямоугольный треугольник

АВС, у которого С =90о, а гипотенуза равна 6 2 см. Через сторону АВ и вершину С1

проведено сечение. Найдите угол между плоскостью сечения и плоскостью основания, если

длина бокового ребра равна 3 см.

А) 45о; Б) arctg 1

2; В) arctg 2; Г) arctg

2

2; Д) arctg 2.

5. В основании прямой призмы АВСА1В1С1 лежит прямоугольный треугольник АВС, у которого <

C = 90о, АС = 4 см, ВС = 3 см. Через сторону АС и вершину В1 проведена плоскость. Угол В1АС равен 60о. Найдите площадь боковой поверхности призмы.

А) 12 39 см2; Б) 35 39 см2; В) 6 39 см2; Г) определить нельзя; Д) 10 39 см2.

6. В наклонной треугольной призме с боковым ребром, равным 10 см, площади двух граней равны

70 см2 и 150 см2, угол между ними – 60о. Найдите площадь боковой поверхности призмы.

А) 367,5 3 см2; Б) 350 см2; В) определить нельзя; Г) 262,5 3 см2; Д) 90 см2.

7. Основанием пирамиды МАВС служит прямоугольный треугольник АВС, у которого < C =

90о, < A = 30о, ВС = 6 см. Боковые ребра наклонены к основанию под углом 60о. Найдите высоту пирамиды.

А) 6 3 см; Б) 6 2 см; В) 6 см; Г) 3 2 см; Д) 3 см.

8. В правильной четырехугольной пирамиде высота равна 4 см, а длина диагонали основания –

6 2 см. Найдите площадь полной поверхности пирамиды.

А) 96 см2; Б) 156 см2; В) 36 см2; Г) 60 см2; Д) 150 см2. 9. В правильной треугольной усеченной пирамиде стороны оснований равны 6 см и 3 см. Высота

усеченной пирамиды равна 13

2 см. Найдите площадь боковой поверхности усеченной

пирамиды. А) 18 см2; Б) 9 см2; В) 36 см2; Г) 72 см2; Д) 27 см2.

Page 33: 1. Пояснительная запискаjar.edunoskol.ru/images/dokument/RAB_PROG/geometria_coo.pdf · Стандарт основного общего образования

10. Найдите угол между диагоналями куба.

А) arccos 1

3; Б) arcsin

1

3; В) arctg

1

3; Г) arcsin

6

3; Д) arccos

6

3.

11. Найдите площадь полной поверхности правильного октаэдра, если его ребро равно 6 см.

А) 36 3 см2; Б) 72 3 см2; В) 12 3 см2; Г) 24 3 см2; Д) 144 3 см2.

12. Правильный тетраэдр и правильный икосаэдр имеют равную площадь полной поверхности.

Определите ребро правильного икосаэдра, если ребро правильного тетраэдра равно 6 см.

А) Определить нельзя; Б) 6 5 см; В) 1,2 5 см; Г) 6 см; Д) 3 5 см.

Зачет № 3 по теме «Многогранники»

Вариант 4

1. Расстояние от вершины верхнего основания куба до центра нижнего основания равно 2 3 см.

Найдите длину диагонали грани куба.

А) 8 см; Б) 4 см; В) 2 2 см; Г) 2 см; Д) 1 см.

2. Дан прямоугольный параллелепипед АВСDA1B1C1D1, у которого BD1 = d, AC = m, AB = n.

Найдите расстояние между плоскостями АВВ1 и DСС1.

А) 𝑑2 − 𝑚2 − 𝑛2; Б) 𝑑2 − 𝑚2; В) 𝑑2 − 𝑛2;

Г) 𝑚2 − 𝑛2; Д) определить нельзя.

3. Измерения прямоугольного параллелепипеда равны 3 м, 4 м и 5 м. Определите угол между

диагональю параллелепипеда и плоскостью основания.

А) 45о; Б) arctg 3

5; В) arctg

4

5 ; Г) arcsin

3 2

5 ; Д) определить нельзя.

4. В основании прямой призмы АВСА1В1С1 лежит равнобедренный прямоугольный треугольник

АВС, у которого < С = 90о. Через сторону АВ и вершину С1 проведено сечение, составляющее угол 60о с плоскостью основания. Найдите длину АВ, если длина бокового ребра равна 3 см.

А) Определить нельзя; Б) 3 см; В) 2 3 см; Г) 3 3 см; Д) 1 см.

5. В основании прямой призмы АВСА1В1С1 лежит прямоугольный треугольник АВС, у которого <

C = 90о, АС = 5 см. Через сторону ВС и вершину А1 проведена плоскость. Найдите площадь боковой поверхности призмы, если < ВА1С = 30о, ВА1 = 10 см.

А) 50 2 + 1 см2; Б) 50 2 см2; В) определить нельзя; Г) 50 см2; Д) 50 3 см2.

6. В наклонной треугольной призме с боковым ребром, равным 5 см, площади двух граней равны

15 см2 и 25 см2, угол между ними равен 120о. Найдите площадь боковой поверхности призмы.

А) 3

4 3 см2; Б) определить нельзя; В) 30 см2; Г) 15 см2; Д) 75 см2.

7. В основании пирамиды МАВС лежит треугольник АВС, у которого < АСВ = 150о, ВА = 6 см. Боковые ребра наклонены к основанию под углом 45о. Найдите высоту пирамиды.

А) 6 см; Б) 12 см; В) 2 3 см; Г) 4 3 см; Д) 3 2 см.

8. Высота правильной треугольной пирамиды равна 12 см, высота основания – 15 см. Найдите

площадь полной поверхности пирамиды.

А) 75 3 см2; Б) 195 3 см2; В) 270 3 см2; Г) 810 см2; Д) 120 3 см2.

9. В правильной четырехугольной усеченной пирамиде стороны оснований равны 8 см и 10 см.

Высота усеченной пирамиды равна 3 см. Найдите площадь боковой поверхности усеченной

пирамиды.

Page 34: 1. Пояснительная запискаjar.edunoskol.ru/images/dokument/RAB_PROG/geometria_coo.pdf · Стандарт основного общего образования

А) 72 см2; Б) 36 см2; В) 24 см2; Г) 108 см2; Д) 18 см2.

10. Найдите угол между диагональю куба и плоскостью его основания.

А) 2 arcsin 3

3 ; Б) arcos

3

3 ; В) arcsin

2 3

3 ; Г) 2 arcos

3

3 ; Д) arcsin

3

3 .

11. Найдите площадь полной поверхности правильного октаэдра, если его ребро равно 3 см.

А) 36 3 см2; Б) 72 3 см2; В) 12 3 см2; Г) 18 3 см2; Д) 144 3 см2.

12. Правильный тетраэдр и правильный октаэдр имеют равную площадь полной поверхности. Определите ребро правильного тетраэдра, если ребро правильного октаэдра равно 3 см.

А) Определить нельзя; Б) 6 2 см; В) 0,4 2 см; Г) 3 см; Д) 3 2 см.

Геометрия 10 класс

Ответы к зачету № 3

по теме «Многогранники»

Вариант Задания

1 2 3 4 5 6 7 8 9 10 11 12

I Б В Г А В В Б Д Г В Б Г

II В В А Б А Г Б А Д Г Б Б

III Д Г Б Г А Б А А Д А Б В

IV Б Г А В А Д А В А Д Г Д

Геометрия – 10 класс

Зачет № 3 по теме «Многогранники»

1. Начертите пирамиду и назовите ее элементы.

2. Начертите призму и назовите ее элементы.

3. Какая призма называется прямой?

4. Какая призма называется правильной?

5. Какая пирамида называется правильной?

6. Какая пирамида называется усеченной?

7. Какие многогранники называются правильными?

8. Перечислите известные вам виды правильных многогранников.

9. Приведите примеры многогранников, имеющих ось симметрии.

10. Приведите примеры многогранников, имеющих плоскость симметрии.

11. Чему равна площадь боковой поверхности призмы?

12. Чему равна площадь боковой поверхности правильной пирамиды?

13. Чему равна площадь боковой поверхности правильной усеченной пирамиды?

14. Какие точки называются симметричными относительно плоскости?

15. Какие точки называются симметричными относительно прямой?

16. Чему равна площадь полной поверхности призмы?

Решите задачи:

Page 35: 1. Пояснительная запискаjar.edunoskol.ru/images/dokument/RAB_PROG/geometria_coo.pdf · Стандарт основного общего образования

1. В правильной шестиугольной призме сторона основания 4 см, боковое ребро 7 см.

Найдите площадь полной поверхности призмы.

2. В основании прямой четырехугольной призмы – ромб. Высота призмы равна 8 см,

сторона основания 5 см. Найдите площадь боковой поверхности призмы.

3. Площадь боковой поверхности прямой треугольной призмы равна 72 см2, стороны

основания призмы 6 см, 4 см и 7 см. Найдите длину бокового ребра.

4. В прямоугольном параллелепипеде стороны основания 6 см и 8 см, а диагональ

наклонена к плоскости основания под углом 60о. Определите длину бокового

ребра.

5. В правильной треугольной пирамиде боковое ребро равно 9 см, а высота 6 см.

Найдите длину стороны основания.

6. В основании пирамиды – прямоугольник, стороны основания которого равна 8 см и

6 см. Высота пирамиды проходит через точку пересечения диагоналей основания и

равна 10 см. Определите площади диагональных сечений пирамиды.

7. Высота правильной треугольной пирамиды равна 5 см, а сторона основания 8 см.

Найдите длину бокового ребра.

8. Основание пирамиды – квадрат. Одно из боковых ребер перпендикулярно

основанию Наибольшее боковое ребро равно 10 см и наклонено к плоскости

основания под углом 60о. Найдите сторону квадрата.

Геометрия 11 класс

Входной тест

(Итоговый тест за 10 класс)

Вариант 1

1. Плоскость 𝛼, параллельная стороне АВ треугольника АВС, пересекает его в точках А1 и

В1, лежащих на сторонах АС и ВС соответственно. Найдите А1С, если АС = 15 см,

А1В1 = 4 см, АВ = 20 см.

А. 3 см Б. 4 см В. 10 см Г. 12 см Д. 7,5 см

2. Найдите расстояние от некоторой точки до плоскости квадрата, если расстояние от этой

точки до всех его сторон равно 4 см, а сторона квадрата равна 2 см.

А. 13 см Б. 2 3 см В. 15 см Г. 17 см Д. 3 2 см

3. Выберите верное утверждение.

А. Если плоскость пересекает одну из параллельных плоскостей, то она не пересекает

другую.

Б. Противоположные ребра тетраэдра лежат на пересекающихся прямых.

В. Если прямые скрещиваются, то расстояние между ними не определить.

Г. Все грани правильной усеченной пирамиды – прямоугольные трапеции.

Д. Проекцией точки на плоскость называется основание перпендикуляра, проведенного из

этой точки к плоскости, если точка не лежит в плоскости, и сама точка, если она лежит в

плоскости.

4. Основанием пирамиды является параллелограмм со сторонами 3 и 7 и одной из

диагоналей, равной 6. Высота пирамиды равна 4, ее основанием является точка

пересечения диагоналей параллелограмма, лежащего в основании. Найдите боковые

ребра пирамиды.

А. 5, 5, 5, 6 Б. 5, 5, 6, 6 В. 5, 6, 6, 6 Г. 5, 5, 5, 5 Д. 6, 6, 6, 6

Page 36: 1. Пояснительная запискаjar.edunoskol.ru/images/dokument/RAB_PROG/geometria_coo.pdf · Стандарт основного общего образования

5. Основанием пирамиды DABC является треугольник со сторонами АС = 13, АВ = 15,

СВ = 14. Боковое ребро DA, равное 9, перпендикулярно к плоскости основания.

Найдите площадь полной поверхности пирамиды.

А. 273 Б. 630 В. 231 Г. 315 Д. 356

6. Найдите площадь полной поверхности правильной треугольной призмы АВСА1В1С1,

если АВ = 2 см, АА1 = 1 см.

А. ( 3 + 6) см2 Б. (2 3 + 3) см

2 В. ( 3 + 3) см

2 Г. (2 3 + 6) см

2 Д. (4 3 +

6) см2

7. В правильной треугольной призме АВСА1В1С1 АВ = 2 см, АА1 = 1 см. Найдите угол,

который составляет прямая АВ1 с плоскостью АВС.

А. 𝑎𝑟𝑐𝑡𝑔 0,5 Б. 𝑎𝑟𝑐𝑡𝑔 2 В. 𝑎𝑟𝑐𝑡𝑔 2 Г. 𝑎𝑟𝑐𝑡𝑔 0,5 2 Д. 45о

8. Найдите площадь сечения правильной треугольной призмы АВСА1В1С1 плоскостью

АСВ1 при условии, что АВ = 2 см, АА1 = 1 см.

А. 4 см2 Б. 1 см

2 В. 6 см

2 Г. 8 см

2 Д. 2 см

2

9. Дана правильная треугольная призма АВСА1В1С1, у которой АВ = 2 см, АА1 = 1 см.

Найдите угол между плоскостями АВ1С и АВС.

А. 60о Б. 45

о В. 30

о Г. 120

о Д. 90

о

10. Упростите выражение АС + ВВ1 + ВА + 𝐷1В + В1𝐷1

+ 𝐷𝐶 , если АВСДА1В1С1D1 –

параллелепипед.

А. АС Б. 0 В. ВВ1 Г. 𝐷С Д. ВА

Геометрия 11 класс

Входной тест

(Итоговый тест за 10 класс)

Вариант 2

1. Плоскость 𝛼, параллельная стороне АВ треугольника АВС, пересекает его в точках А1 и

В1, лежащих на сторонах АС и ВС соответственно. Найдите А1А, если А1С = 5 см,

А1В1 = 7 см, АВ = 21 см.

А. 12 см Б. 10 см В. 15 см Г. 21 см Д. 5 см

2. Расстояние от некоторой точки до плоскости квадрата равно 3 см. Сторона квадрата

равна 4 см. Найдите расстояние от этой точки до всех его вершин, если вершины

равноудалены от нее.

А. 4 3 см Б. 15 см В. 17 см Г. 24 см Д. 3 2 см

3. Выберите верное утверждение.

А. Если плоскость пересекает одну из параллельных прямых, то она не пересекает другую.

Б. Противоположные ребра тетраэдра лежат на параллельных прямых.

В. Наклонная всегда меньше перпендикуляра, если они проведены из одной точки.

Г. Все грани правильной треугольной призмы – правильные треугольники.

Д. Прямая, проведенная в плоскости через основание наклонной перпендикулярно к ней,

перпендикулярна и к ее проекции.

Page 37: 1. Пояснительная запискаjar.edunoskol.ru/images/dokument/RAB_PROG/geometria_coo.pdf · Стандарт основного общего образования

4. Площадь сечения правильной треугольной призмы, проведенного через боковое ребро и

середину противолежащей стороны нижнего основания, равна 2 3 см2. Найдите длину

ребра этой призмы при условии, что все ее ребра равны.

А. 2 см Б. 1 см В. 4 см Г. 3 см Д. Определить

нельзя

5. В прямом параллелепипеде ABCDA1B1C1D1 АВ = 2, AD = 3 2, ∠BAD = 45о, В1D =

19. Найдите площадь полной поверхности параллелепипеда.

А. 18 2 + 12 Б. 3 2 + 2 В. 3 2 + 24 Г. 18 2 + 24 Д.

24 2 + 18

6. Найдите площадь полной поверхности правильной четырехугольной пирамиды

ЕАВСD, если АE = 2 2 см, АВ = 2 см.

А. ( 7 + 1) см2 Б. (4 7 + 1) см

2 В. ( 7 + 4) см

2 Г. (4 7 + 4) см

2 Д. 4 7 см

2

7. В правильной четырехугольной пирамиде ЕАВСD АЕ = 2 2 см, АВ = 2 см. Найдите

угол, который составляет прямая ЕС с плоскостью АВС.

А. 45о Б. 60

о В. 30

о Г. 120

о Д. 90

о

8. Найдите площадь сечения правильной четырехугольной пирамиды ЕАВСD плоскостью

АЕС при условии, что АЕ = 2 2 см, АВ = 2 см.

А. 1 см2 Б. 2 см

2 В. 2 2 см

2 Г. 3 см

2 Д. 2 3 см

2

9. Дана правильная четырехугольная пирамида ЕАВСD, у которой АЕ = 2 2 см, АВ = 2

см. Найдите угол между плоскостями ЕВС и АВС.

А. 𝑎𝑟𝑐𝑐𝑡𝑔 7 Б. arcsin 7

7 В. arccos

7

7 Г. 𝑎𝑟𝑐𝑡𝑔 7

Д. 45о

10. Упростите выражение 𝐵1𝐷1 + 𝐶1𝐶 + 𝐶1𝐵 + 𝐴𝐶1

+ 𝐶𝐴 + 𝐴1𝐷1 , если АВСДА1В1С1D1 –

параллелепипед.

А. 𝐵1𝐴1 Б. 0 В. 𝐶𝐶1

Г. С𝐴 Д. 𝐵1𝐶

Ответы к входному тесту по геометрии в 11 классе

№ задания 1 2 3 4 5 6 7 8 9 10

Вариант 1 А В Д Б Г Г А Д В А

Вариант 2 Б В Д А Г Г Б Д В Д

Page 38: 1. Пояснительная запискаjar.edunoskol.ru/images/dokument/RAB_PROG/geometria_coo.pdf · Стандарт основного общего образования

Геометрия 11 класс

Контрольная работа № 1

по теме «Метод координат в пространстве»

Вариант 1

1. Вычислите скалярное произведение векторов 𝑚 и 𝑛 , если 𝑚 = 𝑎 + 2𝑏 − 𝑐, 𝑛 = 2𝑎 −

𝑏, 𝑎 = 2, 𝑏 = 3,∧

𝑎 𝑏 = 60°, 𝑐 ⊥ 𝑎 , 𝑐 ⊥ 𝑏 .

2. Дан куб ABCDA1B1C1D1. Найдите угол между прямыми AD1 и ВМ, где М – середина

ребра DD1.

3. При движении прямая 𝑎 отображается на прямую 𝑎1 , а плоскость - на плоскость 1.

Докажите, что если 𝑎 , то 𝑎1 1 .

Геометрия 11 класс

Контрольная работа № 1

по теме «Метод координат в пространстве»

Вариант 2

1. Вычислите скалярное произведение векторов 𝑚 и 𝑛 , если 𝑚 = 2𝑎 − 𝑏 + 𝑐, 𝑛 = 𝑎 −

2𝑏, 𝑎 = 3, 𝑏 = 2,∧

𝑎 𝑏 = 60°, 𝑐 ⊥ 𝑎 , 𝑐 ⊥ 𝑏 .

2. Дан куб ABCDA1B1C1D1. Найдите угол между прямыми AС и DС1.

3. При движении прямая 𝑎 отображается на прямую 𝑎1 , а плоскость - на плоскость 1.

Докажите, что если 𝑎 , то 𝑎1 1 .

еометрия 11 класс

Контрольная работа № 2

по теме «Цилиндр, конус и шар»

Вариант 1

1. Осевое сечение цилиндра – квадрат, площадь основания цилиндра равна 16π см2. Найдите

площадь полной поверхности цилиндра.

2. Высота конуса равна 6 см, угол при вершине осевого сечения равен 120о. Найдите:

а) площадь сечения конуса плоскостью, проходящей через две образующие, угол

между которыми равен 30о;

б) площадь боковой поверхности конуса.

3. Диаметр шара равен 2т. Через конец диаметра проведена плоскость под углом 45о

к нему. Найдите длину линии пересечения сферы этой плоскостью.

Page 39: 1. Пояснительная запискаjar.edunoskol.ru/images/dokument/RAB_PROG/geometria_coo.pdf · Стандарт основного общего образования

Геометрия 11 класс

Контрольная работа № 2

по теме «Цилиндр, конус и шар»

Вариант 2

1. Осевое сечение цилиндра – квадрат, диагональ которого равна 4 см. Найдите площадь

полной поверхности цилиндра.

2. Радиус основания конуса равен 6 см, а образующая наклонена к плоскости основания

под углом 30о. Найдите:

а) площадь сечения конуса плоскостью, проходящей через две образующие, угол

между которыми равен 60о;

б) площадь боковой поверхности конуса.

3. Диаметр шара равен 4т. Через конец диаметра проведена плоскость под углом 30о к

нему. Найдите площадь сечения шара этой плоскостью.

Геометрия 11 класс

Контрольная работа № 3

по теме «Объѐмы тел»

Вариант 1

1. Диаметр шара равен высоте конуса, образующая которого составляет с плоскостью

основания угол 60о. Найдите отношение объѐмов конуса и шара.

2. Объѐм цилиндра равен 96π см3, площадь его осевого сечения – 48 см

2. Найдите

площадь сферы, описанной около цилиндра.

Геометрия 11 класс

Контрольная работа № 3

по теме «Объѐмы тел»

Вариант 2

1. В конус, осевое сечение которого есть правильный треугольник, вписан шар. Найдите

отношение площади сферы к площади боковой поверхности конуса.

2. Диаметр шара равен высоте цилиндра, осевое сечение которого есть квадрат. Найдите

отношение объѐмов шара и цилиндра.

Зачет № 1 по теме «Векторы в пространстве»

Page 40: 1. Пояснительная запискаjar.edunoskol.ru/images/dokument/RAB_PROG/geometria_coo.pdf · Стандарт основного общего образования

Вариант 1

1. Какое из следующих утверждений неверно?

А) Длиной ненулевого вектора АВ называется длина отрезка АВ.

Б) Нулевой вектор считается сонаправленным любому вектору.

В) АВ + ВС = АС .

Г) Разностью векторов 𝑎 и 𝑏 называется такой вектор, сумма которого с вектором 𝑏 равна вектору 𝑎 .

Д) Векторы называются равными, если равны их длины.

2. Упростите выражение: АС + ВВ1 + ВА + 𝐷1В + В1𝐷1

+ 𝐷С , если

АВСDА1В1С1D1 - параллелепипед.

А) АС ;

Б) 0 ;

В) ВВ1 ;

Г) 𝐷𝐶 ;

Д) ВА .

3. Укажите вектор 𝑥 в тетраэдре АВСD, если АС = АВ − 𝑥 − 𝐶𝐷.

А) 𝐵𝐷 ;

Б) АВ ;

В) 𝐷𝐶 ;

Г) 𝐷𝐵 ;

Д) 𝐶𝐷 .

4. Векторы −𝐷𝐸 + 𝐷𝐹 − 𝐾𝐹 и МС − МК − ЕС являются:

А) равными;

Б) противоположными;

В) сонаправленными;

Г) нулевыми;

Д) коллинеарными.

5. Какое из следующих утверждений верно?

А) Сумма нескольких векторов зависит от того, в каком порядке они складываются.

Б) Противоположные векторы равны.

В) Для нахождения разности векторов необходимо, чтобы они выходили из одной точки.

Г) Произведением вектора на число является число.

Д) Для любых векторов 𝑎 и 𝑏 не выполняется равенство 𝑎 + 𝑏 = 𝑏 + 𝑎 .

6. Какое из следующих утверждений неверно?

А) Векторы называются компланарными, если при откладывании их от одной и той же

точки они будут лежать в одной плоскости.

Б) Если вектор 𝑐 можно разложить по векторам 𝑎 и 𝑏 , т.е. представить в виде 𝑐 = 𝑥𝑎 +

𝑦𝑏 , где 𝑥, 𝑦 - некоторые числа, то векторы 𝑎 , 𝑏 и 𝑐 компланарны.

В) Для сложения трех некомпланарных векторов используют правило параллелепипеда.

Г) Любые два вектора компланарны.

Д) Любые три вектора некомпланарны.

Page 41: 1. Пояснительная запискаjar.edunoskol.ru/images/dokument/RAB_PROG/geometria_coo.pdf · Стандарт основного общего образования

7. АВСDА1В1С1D1 - параллелепипед. Какой из предложенных векторов будет

компланарен с векторами АВ1 и АС ?

А) ВВ1 ;

Б) С1В1 ;

В) 𝐷В1 ;

Г) СВ1 ;

Д) СС1 .

8. Векторы 𝑝 , 𝑎 , 𝑏 некомпланарны, если:

А) при откладывании из одной точки они не лежат в одной плоскости;

Б) два из данных векторов коллинеарны;

В) один из данных векторов нулевой;

Г) 𝑝 = 𝑎 − 𝑏 ; Д) 𝑝 = 𝑎 .

9. Известно, что 2АС = АВ + А𝐷 , тогда векторы АС , АВ , А𝐷 являются:

А) некомпланарными;

Б) сонаправленными;

В) коллинеарными;

Г) нулевыми;

Д) компланарными.

10. Даны параллелограммы АВСD и АВ1С1D1. Тогда векторы ВВ1 , СС1

, 𝐷𝐷1 :

А) нулевые;

Б) равные;

В) противоположные;

Г) компланарные;

Д) некомпланарные.

Геометрия 11 класс

Зачет № 1 по теме «Векторы в пространстве»

Вариант 2

1. Какое из следующих утверждений неверно?

А) Длиной нулевого вектора АВ называется длина отрезка АВ.

Б) Любая точка пространства рассматривается как нулевой вектор.

В) АВ - АС = СВ .

Г) Для любых векторов 𝑎 и 𝑏 выполняется равенство 𝑎 + −𝑏 = 𝑎 − 𝑏 . Д) Векторы называются равными, если они сонаправлены и равны их длины.

2. Упростите выражение: 𝐵1𝐷1 + 𝐶1𝐶 + 𝐶1В + 𝐴𝐶1

+ 𝐶𝐴 + 𝐴1𝐷1 , если

АВСDА1В1С1D1 - параллелепипед.

А) 𝐵1𝐴1 ;

Page 42: 1. Пояснительная запискаjar.edunoskol.ru/images/dokument/RAB_PROG/geometria_coo.pdf · Стандарт основного общего образования

Б) 0 ;

В) 𝐶𝐶1 ;

Г) 𝐶𝐴 ;

Д) 𝐵1𝐶 .

3. Укажите вектор 𝑥 в тетраэдре АВСD, если С𝐷 = 𝑥 − 𝐷𝐵 − 𝐴𝐶.

А) 𝐷𝐵 ;

Б) 𝐶𝐷 ;

В) 𝐵𝐴 ;

Г) 𝐴𝐵 ;

Д) 𝐵𝐷 .

4. Векторы 𝐴𝐶1 - 𝐴𝐶 + 𝐴1𝐶1

и 𝐴1𝐴 − 𝐶𝐵 + 𝐴𝐵 являются:

А) противоположными;

Б) коллинеарными;

В) сонаправленными;

Г) нулевыми;

Д) равными.

5. Какое из следующих утверждений верно?

А) Разностью векторов 𝑎 и 𝑏 назывется такой вектор, разность которого с вектором 𝑏 равна вектору 𝑎 .

Б) Если векторы 𝑎 и 𝑏 коллинеарны и 𝑎 ≠ 0 , то существует такое число k, что 𝑏 = k𝑎 .

В) Векторы называются равными, если они сонаправлены.

Г) Два вектора, коллинеарные ненулевому вектору, сонаправлены.

Д) Для любых векторов 𝑎 и 𝑏 выполняется равенство 𝑎 (с + 𝑏 ) = 𝑏 с + 𝑎 с .

6. Какое из следующих утверждений неверно?

А) Три вектора будут компланарными, если один из них нулевой.

Б) Если вектора𝑎 , 𝑏 и 𝑐 компланарны, то вектор 𝑐 можно разложить по векторам 𝑎 и 𝑏 ,

т.е. представить в виде 𝑐 = 𝑥𝑎 + 𝑦𝑏 , где 𝑥, 𝑦 - некоторые числа.

В) Для сложения трех компланарных векторов не используют правило параллелепипеда.

Г) Любые два вектора некомпланарны.

Д) Три нулевых вектора компланарны.

7. АВСDА1В1С1D1 - параллелепипед. Какой из предложенных векторов будет

компланарен с векторами СВ1 и АА1

?

А) 𝐶𝐷 ;

Б) 𝐴1В1 ;

В) 𝐴В1 ;

Г) С𝐷1 ;

Д) С𝐵 .

8. Векторы 𝑝 , 𝑎 , 𝑏 компланарны, если:

А) при откладывании из одной точки они не лежат в одной плоскости;

Б) два из данных векторов равны;

В) если любой вектор можно разложить по данным векторам;

Page 43: 1. Пояснительная запискаjar.edunoskol.ru/images/dokument/RAB_PROG/geometria_coo.pdf · Стандарт основного общего образования

Г) если их сумму можно найти с помощью правила параллелепипеда;

Д) если их длины являются измерениями параллелепипеда.

9. Известно, что 2АС = −АВ + 2А𝐷 , тогда векторы АС , АВ , А𝐷 : А) компланарны;

Б) некомпланарны;

В) коллинеарны;

Г) сонаправлены;

Д) нулевые.

10. Даны параллелограммы АВСD и АВ1С1D1. Тогда векторы В1В , С1С , 𝐷1𝐷 : А) нулевые;

Б) равные;

В) компланарные;

Г) некомпланарные;

Д) противоположные.

Геометрия 11 класс

Зачет № 1 по теме «Векторы в пространстве»

Вариант 3

1. Дана правильная треугольная пирамида DABC, сторона основания которой равна

3. Боковые ребра наклонены к основанию под углом 60о. Найдите 𝐷𝐴 + 𝐶𝐵 + 𝐴𝐶 .

А) 1;

Б) 2;

В) 3;

Г) 5;

Д) 6.

2. Ребро куба АВСDА1В1С1D1 равно 1. Найдите 𝐷𝐶1 − 𝐷𝐴1

. А) 1;

Б) 2;

В) 2;

Г) 3;

Д) 0,5 2.

3. Диагонали параллелепипеда АВСDА1В1С1D1 пересекаются в точке О. При каком

значении k справедливо соотношение 𝐴𝐵 + 𝐵1𝐶1 + 𝐶𝑂 = 𝑘𝐶1𝐴 ?

А) – 0,5;

Б) 0,5;

В) 1;

Г) – 1;

Д) ни при каком.

Page 44: 1. Пояснительная запискаjar.edunoskol.ru/images/dokument/RAB_PROG/geometria_coo.pdf · Стандарт основного общего образования

4. В параллелепипеде АВСDА1В1С1D1 А1С1 пересекает В1D1 в точке М, В1𝐷1 =

𝑥𝐷1М . Найдите 𝑥.

А) 0,5;

Б) – 0,5;

В) 1;

Г) – 1;

Д) – 2.

5. В правильной треугольной пирамиде DABC отрезок DO – высота.

𝐷𝑂 = 𝑥𝐷𝐴 + 𝑦𝐷𝐵 + 𝑧𝐷𝐶 . Найдите 𝑥, 𝑦, 𝑧. А) 𝑥 = 0,5, 𝑦 = 𝑧 = 0,25; Б) 𝑥 = 0,5, 𝑦 = 𝑧 = −0,25; В) 𝑥 = 𝑦 = 𝑧 = 0,5; Г) 𝑥 = 0,25; 𝑦 = 𝑧 = 0,5; Д) 𝑥 = 𝑦 = 𝑧 = 0,25.

6. Известно, что АС = 𝑥𝐴𝐵 + 𝑦𝐴𝐷 . Тогда прямые АС и ВD:

А) параллельны;

Б) пересекаются;

В) скрещиваются;

Г) совпадают;

Д) выполняются все условия пунктов А – Г.

7. Даны векторы 𝑚 = 𝑎 − 𝑏 + 𝑐 , 𝑛 = 2𝑎 − 𝑏 + 2𝑐 , 𝑝 = 3𝑎 − 4𝑏 + 𝑐 ,

𝑘 = 3𝑎 − 2𝑏 + 3𝑐 . Укажите тройку компланарных векторов.

А) 𝑚 , 𝑛 , 𝑘 ;

Б) 𝑚 , 𝑛 , 𝑝 ;

В) 𝑝 , 𝑛 , 𝑘 ;

Г) таких троек нет;

Д) определить нельзя.

8. Дана пирамида PABCD, в основании которой лежит параллелограмм ABCD.

Разложите вектор 𝑃𝐷 по векторам 𝑃𝐴 , 𝑃𝐵 , 𝑃𝐶 .

А) 𝑃𝐶 − 𝑃𝐴 − 𝑃𝐵 ;

Б) 𝑃𝐶 + 𝑃𝐴 + 𝑃𝐵 ;

В) 𝑃𝐶 + 𝑃𝐴 − 𝑃𝐵 ;

Г) 𝑃𝐶 − 𝑃𝐴 + 𝑃𝐵 ;

Д) 𝑃𝐵 + 𝑃𝐴 − 𝑃𝐶 .

9. В тетраэдре ABCD медианы основания BCD пересекаются в точке О. Тогда

вектор АО равен:

А) 1

3 𝐴𝐵 + 𝐴𝐶 + 𝐴𝐷 ;

Б) 1

3 𝐴𝐵 − 𝐴𝐶 + 𝐴𝐷 ;

В) −1

3 𝐴𝐵 + 𝐴𝐶 + 𝐴𝐷 ;

Г) 1

3 𝐴𝐵 + 𝐴𝐶 − 𝐴𝐷 ;

Page 45: 1. Пояснительная запискаjar.edunoskol.ru/images/dokument/RAB_PROG/geometria_coo.pdf · Стандарт основного общего образования

Д) 1

3 𝐴𝐵 − 𝐴𝐶 − 𝐴𝐷 .

10. Даны векторы 𝑐 = 2𝑎 − 3𝑏 , 𝑑 = 3𝑎 + 2𝑏 , 𝑝 = 2𝑐 − 3𝑑. Найдите

коэффициенты 𝑥, 𝑦 разложения вектора 𝑝 по векторам 𝑎 и 𝑏 . А) 𝑥 = 13, 𝑦 = 0; Б) 𝑥 = −5, 𝑦 = −12; В) 𝑥 = 5, 𝑦 = −12; Г) 𝑥 = −5, 𝑦 = 0; Д) 𝑥 = 15, 𝑦 = 12.

Геометрия 11 класс

Зачет № 1 по теме «Векторы в пространстве»

Вариант 4

1. Основанием пирамиды МABC служит прямоугольный треугольник АВС

(< С = 90о); АС = 6; ВС = 8. Боковые ребра наклонены к основанию под углом 60о.

Найдите 𝐴С + 𝐵М + 𝐶В .

А) 6;

Б) 10;

В) 8;

Г) 5 3;

Д) 5.

2. В правильной треугольной призме АВСА1В1С1 сторона основания равно 1,

точка Е – середина А1С1. Найдите СЕ − СВ1 .

А) 1;

Б) 2;

В) 3;

Г) 3;

Д) 0,5 3.

3. Диагонали параллелепипеда АВСDА1В1С1D1 пересекаются в точке О. При каком

значении k справедливо соотношение 𝑘(𝐶𝐷 + 𝐷𝐴 + 𝐴𝑂 ) = 𝐴1𝐶 ?

А) 2;

Б) – 2;

В) 1;

Г) – 1;

Д) ни при каком.

4. В параллелепипеде АВСDА1В1С1D1 А1С пересекает В1D в точке М, 𝐴1𝐶 =

𝑥𝐶М . Найдите 𝑥.

А) 0,5;

Б) – 0,5;

В) 1;

Г) – 1;

Д) – 2.

Page 46: 1. Пояснительная запискаjar.edunoskol.ru/images/dokument/RAB_PROG/geometria_coo.pdf · Стандарт основного общего образования

5. В тетраэдре DABC медианы DE и CF грани DBC пересекаются в точке О.

А𝐷 = 𝑥𝐴О + 𝑦АС + 𝑧АВ . Найдите 𝑥, 𝑦, 𝑧. А) 𝑥 = 𝑦 = −1, 𝑧 = 3; Б) 𝑥 = 𝑧 = −1, 𝑦 = 3; В) 𝑥 = 𝑦 = 𝑧 = 3; Г) 𝑥 = 3; 𝑦 = 𝑧 = −1; Д) 𝑥 = 𝑦 = 𝑧 = −1.

6. Известно, что А𝐵 = 𝑥𝐴𝐶 + 𝑦𝐴𝐷 . Тогда прямые АB и CD:

А) параллельны;

Б) совпадают;

В) пересекаются;

Г) скрещиваются;

Д) выполняются все условия пунктов А – Г.

7. Даны векторы 𝑚 = 𝑎 − 𝑏 + 𝑐 , 𝑛 = 2𝑎 − 𝑏 + 2𝑐 , 𝑝 = 3𝑎 − 4𝑏 + 𝑐 ,

𝑘 = 𝑎 − 3𝑏 − 𝑐 . Укажите тройку компланарных векторов.

А) определить нельзя;

Б) таких троек нет;

В) 𝑚 , 𝑛 , 𝑘 ; Г) 𝑚 , 𝑛 , 𝑝 ;

Д) 𝑝 , 𝑛 , 𝑘 .

8. Дана пирамида EABCD, в основании которой лежит параллелограмм ABCD.

Разложите вектор 𝐸𝐴 по векторам 𝐸𝐵 , 𝐸𝐶 , 𝐸𝐷 .

А) 𝐸𝐵 + 𝐸𝐶 + 𝐸𝐷 ;

Б) 𝐸𝐵 − 𝐸𝐶 − 𝐸𝐷 ;

В) −𝐸𝐵 + 𝐸𝐶 + 𝐸𝐷 ;

Г) 𝐸𝐵 − 𝐸𝐶 + 𝐸𝐷 ;

Д) 𝐸𝐵 + 𝐸𝐶 − 𝐸𝐷 ;

9. В тетраэдре ABCD медианы основания BCD пересекаются в точке О. Тогда

вектор ОА равен:

А) 1

3 𝐴𝐵 + 𝐴𝐶 + 𝐴𝐷 ;

Б) −1

3 𝐴𝐵 − 𝐴𝐶 + 𝐴𝐷 ;

В) −1

3 𝐴𝐵 − 𝐴𝐶 − 𝐴𝐷 ;

Г) 1

3 𝐴𝐵 + 𝐴𝐶 − 𝐴𝐷 ;

Д) −1

3 𝐴𝐵 + 𝐴𝐶 + 𝐴𝐷 .

10. Даны векторы 𝑐 = 2𝑎 + 3𝑏 , 𝑑 = 3𝑎 − 2𝑏 , 𝑝 = −2𝑐 + 3𝑑. Найдите

коэффициенты 𝑥, 𝑦 разложения вектора 𝑝 по векторам 𝑎 и 𝑏 . А) 𝑥 = 13, 𝑦 = 0; Б) 𝑥 = −5, 𝑦 = −12; В) 𝑥 = 5, 𝑦 = −12; Г) 𝑥 = −5, 𝑦 = 0; Д) 𝑥 = 15, 𝑦 = 12.

Page 47: 1. Пояснительная запискаjar.edunoskol.ru/images/dokument/RAB_PROG/geometria_coo.pdf · Стандарт основного общего образования

Геометрия 11 класс

Ответы к зачету № 1 по теме «Векторы в пространстве»

Вариант Задания

1 2 3 4 5 6 7 8 9 10

I Д А Г Б В Д Г А Д Г

II А Д Г А Б Г Д Б А В

III Б В А Д А Б А В А Б

IV Б Д Б Д Г В Д Г Д В

Геометрия 11 класс

Зачет № 2

по теме «Метод координат в пространстве»

Вариант 1

1. Выберите верное утверждение.

А) Длина вектора 𝑎 𝑥; 𝑦; 𝑧 вычисляется по формуле 𝑎 = 𝑥 + 𝑦 + 𝑧.

Б) Каждая координата вектора равна сумме соответствующих координат его начала и

конца.

В) Вектор называется координатным, если его длина равна единице.

Г) Каждая координата середины отрезка равна полусумме соответствующих координат.

Д) Любая точка пространства имеет положительные координаты.

2. Даны точки А(5; 3; 2), В(3; - 1; - 4). Найдите длину вектора АВ .

А) 2 14; Б) 6 2; В) 8; Г) – 12; Д) 2 3.

3. Даны точки А(- 1; 2; 3) и В(1; - 1; 4). Разложите вектор АВ по координатным векторам.

А) АВ = −2𝑖 + 3𝑗 − 𝑘 ; Б) АВ = 0𝑖 + 𝑗 + 7𝑘 ; В) АВ = 2𝑖 − 3𝑗 + 𝑘 ;

Г) АВ = 3𝑖 − 2𝑗 − 𝑘 ; Д) АВ = 𝑖 − 3𝑗 + 2𝑘 .

4. Выберите неверное утверждение.

А) Если у векторов координаты равны, то векторы равны.

Б) Если вектор 𝑎 имеет координаты 𝑎 𝑚; 𝑛; 𝑝 , то его разложение по координатным

векторам будет таким: 𝑎 = 𝑛𝑖 + 𝑝𝑗 + 𝑚𝑘 .

В) Каждая координата разности двух векторов равна разности соответствующих

координат этих векторов.

Г) Любая точка пространства имеет три координаты.

Д) Расстояние между точками М1(𝑥1; 𝑦1 ; 𝑧1) и М2(𝑥2; 𝑦2; 𝑧2) вычисляется по формуле

𝑑 = (𝑥2 − 𝑥1)2 + (𝑦2 − 𝑦1)2 + (𝑧2 − 𝑧1)2.

5. Найдите угол между векторами 𝑗 и 𝑚 = 2𝑖 − 3𝑘 .

А) 0о; Б) определить нельзя; В) 45

о; Г) 90

о; Д) 180

о.

Page 48: 1. Пояснительная запискаjar.edunoskol.ru/images/dokument/RAB_PROG/geometria_coo.pdf · Стандарт основного общего образования

6. Выберите верное утверждение.

А) Угол между векторами не может быть тупым.

Б) Скалярный квадрат вектора равен квадрату его длины.

В) Скалярное произведение нулевых векторов равно нулю тогда и только тогда, когда эти

векторы перпендикулярны.

Г) Ненулевой вектор называется направляющим вектором прямой, если он лежит на

прямой, перпендикулярной к данной прямой.

Д) Скалярное произведение векторов 𝑎 𝑥; 𝑦; 𝑧 и 𝑏 𝑚; 𝑛; 𝑝 выражается формулой

𝑎 𝑏 = 𝑥𝑝 + 𝑦𝑛 + 𝑧𝑚.

Геометрия 11 класс

Зачет № 2

по теме «Метод координат в пространстве»

Вариант 2

1. Выберите верное утверждение.

А) Расстояние между точками М1(𝑥1; 𝑦1 ; 𝑧1) и М2(𝑥2; 𝑦2 ; 𝑧2) вычисляется по формуле

𝑑 = 𝑥1 − 𝑥2 + 𝑦2 − 𝑦1 + 𝑧2 − 𝑧1 . Б) Каждая координата середины отрезка равна модулю полуразности соответствующих

координат его концов.

В) Только координатные векторы являются единичными.

Г) Каждая координата вектора равна разности соответствующих координат его конца и

начала.

Д) Любая точка пространства имеет неотрицательные координаты.

2. Даны точки А(3; - 1; - 4), В(1; - 4; 2). Найдите длину вектора ВА .

А) 7; Б) 3 5; В) 1; Г) – 3; Д) 3 2.

3. Даны точки А(2; - 1; 3) и В(1; - 2; 1). Разложите вектор ВА по координатным векторам.

А) ВА = −𝑖 − 𝑗 + 2𝑘 ; Б) ВА = 𝑖 − 2𝑗 + 𝑘 ; В) ВА = 𝑖 + 𝑗 + 2𝑘 ;

Г) ВА = 2𝑖 + 𝑗 + 𝑘 ; Д) ВА = −𝑖 − 𝑗 − 2𝑘 .

4. Выберите неверное утверждение.

А) Если векторы равны, то их координаты равны.

Б) Если вектор 𝑎 = 𝑛𝑖 + 𝑝𝑗 + 𝑚𝑘 , то вектор 𝑎 имеет координаты 𝑎 𝑚; 𝑛; 𝑝 .

В) Каждая координата середины отрезка равна полусумме соответствующих координат

его концов.

Г) Любая точка пространства имеет три координаты.

Д) Длина вектора 𝑎 𝑥; 𝑦; 𝑧 вычисляется по формуле 𝑎 = 𝑥2 + 𝑦2 + 𝑧2.

5. Найдите угол между векторами 𝑖 и 𝑛 = −2𝑗 + 𝑘 . А) 180

о; Б) 45

о; В) определить нельзя; Г) 90

о; Д) 0

о.

6. Выберите верное утверждение.

А) Если угол между векторами равен 90о, то векторы называются перпендикулярными.

Page 49: 1. Пояснительная запискаjar.edunoskol.ru/images/dokument/RAB_PROG/geometria_coo.pdf · Стандарт основного общего образования

Б) Скалярным произведением двух векторов называется произведение их длин на синус

угла между ними.

В) скалярный квадрат любого вектора есть число положительное.

Г) От перемены мест сомножителей скалярное произведение изменяется.

Д) Скалярное произведение векторов 𝑎 𝑥; 𝑦; 𝑧 и 𝑏 𝑚; 𝑛; 𝑝 выражается формулой

𝑎 𝑏 = 𝑥𝑛 + 𝑦𝑝 + 𝑧𝑚.

Геометрия 11 класс

Зачет № 2

по теме «Метод координат в пространстве»

Вариант 3

1. На каком расстоянии от плоскости Оху находится точка А(2; - 3; - 5) ?

А) 2; Б) 3; В) 38; Г) 10; Д) 5.

2. Точки А(2; -1; 0) и В(-2; 3; 2) являются концами диаметра окружности. Найдите

координаты центра окружности и ее радиус.

А) О(0; 0; 2), R = 13; Б) О(- 2; 2; 1), R = 2; В) определить нельзя;

Г) О(0; 1; 1), R = 3; Д) О(- 4; 4; 2), R = 5.

3. Точки А(10; - 10; - 2), В(10; - 6; - 2) и С(8; - 6; 0) являются вершинами треугольника.

Вычислите его площадь.

А) 4 2; Б) определить нельзя; В) 64; Г) 4 3; Д) 4 6.

4. Из предложенных векторов выберите некомпланарные векторы.

А) 𝑎 −3; −3; 0 , 𝑖 , 𝑗;

Б) с 1; 0; −2 , 𝑖 , 𝑘;

В) 𝑎 1; −1; 2 , 𝑏 −2; 0; 1 , 𝑐 5; −1; 0 ;

Г) 𝑎 −1; 1; −2 , 𝑏 2; 0; −1 , 𝑐 −5; 1; 0 ;

Д) 𝑏 2; 0; −3 , 𝑖 , 𝑗.

5. Вектор 𝑎 составляет с положительным направлением оси Ох угол 135о. Найдите

абсциссу вектора 𝑎 , если 𝑎 = 2.

А) 2; Б) – 2; В) − 2; Г) 2; Д) определить нельзя.

6. Даны точки А(3; - 2; 4), В(4; - 1; 2), С(6; - 3; 2), D(7; - 3; 1). Найдите угол между

векторами АВ и С𝐷 . А) 150

о; Б) 30

о; В) 45

о; Г) 60

о; Д) 120

о.

Геометрия 11 класс

Зачет № 2

по теме «Метод координат в пространстве»

Page 50: 1. Пояснительная запискаjar.edunoskol.ru/images/dokument/RAB_PROG/geometria_coo.pdf · Стандарт основного общего образования

Вариант 4

1. На каком расстоянии от плоскости Оуz находится точка В(-3; 2; - 5) ?

А) 2; Б) 5; В) 38; Г) 10; Д) 3.

2. Точки А(3; - 4; 2), В(- 3; 2; - 4) и С(1; 3; - 1) являются вершинами треугольника.

Найдите длину медианы, проведенной из точки С.

А) 2 14; Б) 2 5; В) определить нельзя; Г) 17; Д) 3.

3. Вычислите площадь треугольника, вершины которого имеют координаты А(6; 14; - 8),

В(10; - 6; 4), С(2; 6; - 20).

А) 56 10; Б) определить нельзя; В) 62720; Г) 56 35; Д) 56 14.

4. Из предложенных векторов выберите коллинеарные векторы.

А) 𝑎 1; −1; 3 , 𝑏 2; 3; 15 ;

Б) 𝑖 , 𝑘;

В) 𝑚 0; 0; 0 , 𝑛 −2; 9; 1 ; Г) 𝑔 2; 0; −1 , 𝑐 −5; 1; 0 ; Д) 𝑖 , 𝑗.

5. Вектор 𝑎 составляет с положительным направлением оси Оу угол 150о. Найдите

ординату вектора 𝑎 , если 𝑎 = 2 3.

А) Определить нельзя; Б) − 3; ; В) 3; Г) 3;

Д) - 3.

6. Даны точки А(5; - 8; - 1), В(6; - 8; - 2), С(7; - 5; - 11), D(7; - 7; - 9). Найдите угол

между векторами АВ и С𝐷 . А) 120

о; Б) 60

о; В) 45

о; Г) 30

о; Д) 150

о.

Геометрия 11 класс

Ответы к зачету № 2

по теме «Метод координат в пространстве»

Вариант Задания

1 2 3 4 5 6

1 Г А В Б Г Б

2 Г А Б Б Г А

3 Д Г А Д В Б

4 Д Г А В Д А

Геометрия 11 класс

Зачет № 3 по теме «Цилиндр, конус и шар»

Page 51: 1. Пояснительная запискаjar.edunoskol.ru/images/dokument/RAB_PROG/geometria_coo.pdf · Стандарт основного общего образования

Вариант 1

1. Цилиндр нельзя получить вращением:

1) треугольника вокруг одной из сторон;

2) квадрата вокруг одной из сторон;

3) Прямоугольника вокруг одной из сторон.

2. Площадь боковой поверхности цилиндра можно вычислить по формуле:

1) ;2. hRSбок 2) ;2

. hRSбок 3) .. hRSбок

3. Сечением цилиндра плоскостью, перпендикулярной его образующей, является:

1) круг; 2) прямоугольник; 3) трапеция.

4. На основаниях цилиндра взяты две непараллельные друг другу хорды, не проходящие

через центры оснований. Тогда расстояние между хордами:

1) равно высоте цилиндра;

2) больше высоты цилиндра;

3) меньше высоты цилиндра.

5. Боковой поверхностью цилиндра высотой h и диаметром d является квадрат. Тогда

верно, что:

1) ;hd 2) ;dh 3) .dh

6. Развѐрткой боковой поверхности прямого кругового цилиндра может быть:

1) прямоугольник; 2) ромб; 3) параллелограмм.

7. Отношение площадей боковой поверхности и осевого сечения цилиндра равно:

1) ;R 2) ;2 3) .

8. Площадь боковой поверхности цилиндра в 2 раза больше площади основания. Тогда

отношение R

h равно:

1) 1; 2) 2; 3) 3.

9. Концы отрезка АВ лежат на окружностях оснований цилиндра. Тогда

расстояние между прямой АВ и осью цилиндра ОQ равно длине отрезка:

1) ОВ; 2) ON; 3) MN.

10. Конус может быть получен вращением:

1) равностороннего треугольника вокруг его стороны;

2)прямоугольного треугольника вокруг одного из его катетов;

3) прямоугольного треугольника вокруг гипотенузы.

11. Площадь боковой поверхности конуса можно вычислить по формуле:

1) ;. lRSбок 2) ;. hRSбок 3) .. hlSбок

12. Сечением конуса плоскостью, перпендикулярной оси цилиндра,

является:

1) треугольник; 2) прямоугольник; 3) круг.

13. Расстояние от центра основания конуса до плоскости сечения,

проходящей через вершину конуса, равно длине отрезка:

1) ОВ; 2) ОК; 3) ОМ.

14. Развѐрткой боковой поверхности конуса является круговой:

1) сегмент; 2) сектор; 3) слой.

15. Площадь полной поверхности конуса равна:

1) lRSполн 2. ; 2) )(. RlhSполн ; 3) )(. RlRSполн .

16. Наибольший периметр имеет сечение конуса, проходящее через его вершину и хорду,

стягивающую дугу в:

1) ;60 2) ;90

3) .180

Page 52: 1. Пояснительная запискаjar.edunoskol.ru/images/dokument/RAB_PROG/geometria_coo.pdf · Стандарт основного общего образования

17. Если а - образующая конуса, h - высота конуса. Тогда верно, что:

1) ;ha 2) ;ha 3) .ha

18. Через вершину конуса и хорду ВС проведена плоскость. Тогда угол

между этой плоскостью и плоскостью основания – это угол:

1) АВО; 2) АМО; 3) ВАС.

19. Уравнение сферы с центром в точке (3; - 1; 1) и радиусом, равным 4,

имеет вид:

1) ;4)1()1()3( 222 zух

2) ;16)1()1()3( 222 zух

3) .16)1()1()3( 222 zух

20. Центр сферы расположен в плоскости XOZ, тогда:

1) ;8)3()1( 222 zух

2) ;25)2( 222 zух

3) .9)4()5( 222 zух

21. Уравнение сферы .25)5()3( 222 zух Тогда сфера касается плоскости:

1) XOY; 2) YOZ; 3) XOZ.

22. Площадь сферы диаметра d можно вычислить по формуле:

1) ;2

1 2dS 2) ;2dS 3) .2 2dS

23. Плоскость имеет со сферой только одну общую точку, если расстояние от центра

сферы до плоскости:

1) больше еѐ радиуса; 2) меньше еѐ радиуса; 3) равно еѐ радиусу.

24. Можно провести только одну сферу через:

1) одну и ту же окружность;

2) окружность и точку, не принадлежащую ей;

3) четыре точки, являющиеся вершинами ромба.

25. Можно провести бесконечное множество касательных плоскостей к данной сфере

через:

1) прямую, проходящую вне сферы;

2) точку, лежащую вне сферы;

3) точку, лежащую на сфере.

26. Стороны АВ, ВС, АС треугольника АВС касаются сферы с центром О в точках М, N, К

соответственно. Тогда неверно, что:

1) ОА = ОВ = ОС; 2) ОМ = ОN = ОК; 3) ОМ АВ, ON BC, OK AC.

27. Все вершины четырехугольника АВСD принадлежат сфере. Тогда АВСD не может

быть:

1) прямоугольником;

2) равнобедренной трапецией;

3) прямоугольной трапецией.

Геометрия 11 класс

Зачет № 3 по теме «Цилиндр, конус и шар»

Вариант 2

1. Цилиндр можно получить вращением:

1) трапеции вокруг одного из оснований;

Page 53: 1. Пояснительная запискаjar.edunoskol.ru/images/dokument/RAB_PROG/geometria_coo.pdf · Стандарт основного общего образования

2) ромба вокруг одной из диагоналей;

3) прямоугольника вокруг одной из сторон.

2. Площадь боковой поверхности цилиндра можно вычислить по формуле:

1) ;. hRSбок 2)

;2. hRSбок 3) .2 2

. hRSбок

3. Сечением цилиндра плоскостью, перпендикулярной его образующей, является:

1) круг; 2) прямоугольник; 3) трапеция.

4. На основаниях цилиндра взяты две непараллельные друг другу хорды, не проходящие

через центры оснований. Тогда расстояние между хордами:

1) равно образующей цилиндра;

2) больше высоты цилиндра;

3) меньше образующей цилиндра.

5. Боковой поверхностью цилиндра с высотой h и радиусом основания R является

квадрат. Тогда верно, что:

1) ;2R

h 2) ;2

h

R 3) .2Rh

6. Развѐрткой боковой поверхности прямого кругового цилиндра не может быть:

1) прямоугольник; 2) ромб; 3) квадрат.

7. Площадь боковой поверхности цилиндра больше площади осевого сечения цилиндра в:

1)

1 раз; 2) 2 раза; 3) раз.

8. Площадь боковой поверхности цилиндра в 3 раза больше площади основания. Тогда

отношение R

h равно:

1) 1; 2) 1,5; 3)3.

9. Концы отрезка АВ лежат на окружностях оснований цилиндра. АВ не

параллелен OQ. Тогда расстояние между прямой АВ и осью цилиндра

OQ неравно длине отрезка:

1) ОА; 2) MN; 3) ОК.

10. Конус может быть получен вращением:

1) прямоугольного треугольника вокруг гипотенузы;

2) равнобедренного треугольника вокруг медианы, проведенной к основанию;

3) тупоугольного треугольника вокруг одной из его сторон.

11. Площадь боковой поверхности конуса можно вычислить по формуле:

1) ;2

. RSбок 2) ;. lRSбок 3) .. hRSбок

12. Сечением конуса плоскостью, проходящей через вершину конуса и хорду основания,

не может быть:

1) равносторонний треугольник;

2) равнобедренный треугольник;

3) разносторонний треугольник.

13. Расстояние от центра основания конуса до плоскости сечения,

проходящей через вершину конуса, равно длине отрезка:

1) OF; 2) OK; 3) OB.

14. Круговой сектор в градусов является боковой поверхностью конуса с радиусом

основания R и образующей l . Тогда верно, что:

1) ;360 l

R

2) ;

360

2 R

3) .

360

lR

Page 54: 1. Пояснительная запискаjar.edunoskol.ru/images/dokument/RAB_PROG/geometria_coo.pdf · Стандарт основного общего образования

15. Площадь полной поверхности конуса, у которого осевым сечением является

равносторонний треугольник со стороной a , равна:

1) ;4

3 2

. aSполн 2) ;4

32

.

aSполн 3) .3 2

. aSполн

16. Наибольшую площадь имеет сечение конуса, проходящее через его вершину и хорду,

стягивающую дугу в:

1) 60о; 2) 90

о; 3) 180

о.

17. Геометрическим местом точек, равноудалѐнных от всех образующих конуса, является:

1) центр основания конуса;

2) ось конуса;

3) диаметр основания конуса.

18. Через вершину конуса и хорду АВ проведена плоскость. Тогда угол

между этой плоскостью и плоскостью основания – это угол:

1) АСВ; 2) ОАС; 3) СКО.

19. Уравнение сферы с центром в точке (- 2; 1; 1) и радиусом, равным 16,

имеет вид:

1) ;4)1()1()2( 222 zух

2) ;256)1()1()2( 222 zух

3) .256)1()1()2( 222 zух

20. Центр сферы расположен в плоскости YOZ, тогда:

1) ;9)5()1( 222 zух

2) ;7)3( 222 zух

3) .16)6()2( 222 zух

21. Уравнение сферы .25)5()3( 222 zух Тогда сфера касается плоскости:

1) XOY; 2) YOZ; 3) XOZ.

22. Площадь сферы радиуса R можно вычислить по формуле:

1) ;4 2RS 2) ;2RS 3) .2 2RS

23. Сечение сферы есть окружность, если расстояние от центра сферы до плоскости:

1) больше еѐ радиуса; 2) меньше еѐ радиуса; 3) равно еѐ радиусу.

24. Можно провести бесконечное множество сфер через:

1) одну и ту же окружность;

2) окружность и точку, не принадлежащую ей;

3) четыре точки, являющиеся вершинами ромба.

25. Можно провести только одну касательную плоскость к данной сфере через:

1) прямую, проходящую вне сферы;

2) точку, лежащую вне сферы;

3) точку, лежащую на сфере.

26. Все стороны ромба АВСD касаются сферы с центром в точке О. М, N, Р, К – точки

касания. Тогда верно, что:

1) ОА = ОВ = ОС = ОD;

2) ОАС = OBD = OCA = ODB;

3) OM = ON = OP = OK.

27. Все вершины параллелограмма АВСD принадлежат сфере. Тогда АВСD не может

быть:

1) ромбом; 2) квадратом; 3) прямоугольником.

Геометрия 11 класс

Зачет № 3 по теме «Цилиндр, конус и шар»

Page 55: 1. Пояснительная запискаjar.edunoskol.ru/images/dokument/RAB_PROG/geometria_coo.pdf · Стандарт основного общего образования

Вариант 3

1. MN – диаметр основания цилиндра, ММ1 – его образующая. Точка К лежит

на дуге MN. Найдите угол М1КN.

2. Высота цилиндра равна 5 см, а диагональ осевого сечения 13 см. Найдите

длину радиуса основания цилиндра.

3. Высота цилиндра равна 6 см, радиус основания 5 см. На расстоянии 4 см от

оси цилиндра, параллельно ей, проведено сечение. Найдите площадь этого

сечения.

4. Высота цилиндра равна 6 см, радиус основания 5 см. Концы

данного отрезка АВ лежат на окружностях обоих оснований, длина его

равна 10 см. Найдите кратчайшее расстояние от отрезка до оси.

5. Высота цилиндра в 5 раз больше радиуса основания, а площадь полной

поверхности равна 48 см2. Найдите радиус основания цилиндра.

6. Основанием прямой призмы служит треугольник со сторонами 6 см, 8 см и 10 см.

Высота призмы равна 4 см. Найдите площадь боковой поверхности описанного около

призмы цилиндра.

7. Радиус основания конуса равен 5 см, а высота 12 см. Найдите длину

образующей конуса.

8. Осевое сечение конуса – равносторонний треугольник со

стороной 312 см. Найдите высоту конуса.

9. Через вершину конуса под углом 60о к основанию проведена плоскость,

отсекающая четверть окружности основания. Высота конуса равна 32 см.

Найдите площадь сечения.

10. Высота конуса 4 см, радиус основания 3 см. Боковая поверхность конуса развернута на

плоскость. Найдите угол полученного сектора.

11. Диаметр CD и хорда АВ основания конуса соответственно равны 30 см и 18 см.

Тангенс угла между плоскостью основания конуса и плоскостью сечения, проходящего

через вершину конуса и хорду АВ, равен 1,5. Найдите тангенс угла наклона

образующей конуса к плоскости основания.

12. Площадь боковой поверхности усеченного конуса равна 208 см2, длина образующей

13 см, высота – 5 см. Найдите радиус меньшего основания.

13. Даны точки А(1; 2; - 3) и В( 22 ; 0; 3 ). Укажите точку, принадлежащую сфере,

заданной уравнением 11222 zух .

Page 56: 1. Пояснительная запискаjar.edunoskol.ru/images/dokument/RAB_PROG/geometria_coo.pdf · Стандарт основного общего образования

14. Найдите радиус сферы, заданной уравнением .32 222 zухх

15. Через точку А(3, 4; 12), принадлежащую сфере ,169222 zух проведена

плоскость, перпендикулярная оси OZ. Найдите радиус сечения.

16. Радиус шара равен 17 см. Найдите площадь сечения шара плоскостью, находящейся на

расстоянии 15 см от центра.

17. Все стороны правильного треугольника касаются сферы

диаметром 12 см, плоскость треугольника удалена на расстояние

3 см от центра сферы. Найдите сторону треугольника.

18. Все вершины квадрата со стороной 16 см принадлежат сфере.

Расстояние от центра сферы до плоскости квадрата 4 см. Найдите радиус

сферы.

Геометрия 11 класс

Зачет № 3 по теме «Цилиндр, конус и шар»

Вариант 4

1. АВ – диаметр нижнего основания цилиндра, ВВ1 – его образующая. Точка

С лежит на дуге АВ. Найдите угол В1СА.

2. Радиус основания цилиндра 2 см, высота 3 см. Найдите длину диагонали

осевого сечения.

3. Высота цилиндра 8 см, радиус основания 5 см. Цилиндр пересечен

плоскостью параллельно оси так, что в сечении получился квадрат.

Найдите расстояние от этого сечения до оси цилиндра.

4. Концы отрезка АВ длиной 13 см лежат на окружностях

оснований цилиндра. Радиус цилиндра равен 10 см,

расстояние между прямой АВ и осью цилиндра равно 8 см.

Найдите высоту цилиндра.

5. Высота цилиндра в 10 раз больше радиуса основания, а площадь полной поверхности

равна 198 см2. Найдите радиус основания цилиндра.

6. Основанием прямого параллелепипеда служит ромб со сторонами 4 см и

углом в 60о. Площадь боковой поверхности вписанного в параллелепипед

цилиндра равна 310 см2. Найдите высоту параллелепипеда.

Page 57: 1. Пояснительная запискаjar.edunoskol.ru/images/dokument/RAB_PROG/geometria_coo.pdf · Стандарт основного общего образования

7. Радиус основания конуса равен 6 см, длина образующей 10 см. Найдите высоту конуса.

8. Осевое сечение конуса – прямоугольный треугольник с гипотенузой 12 см.

Найдите высоту конуса.

9. Через вершину конуса и хорду АВ, стягивающую дугу в 60о, проведена

плоскость, составляющая с плоскостью основания угол 30о. Высота

конуса 1 см. Найдите площадь сечения.

10. Круговой сектор с углом 200о и радиусом 18 см свѐрнут в коническую поверхность.

Найдите радиус основания получившегося конуса.

11. Диаметр CD и хорда АВ основания конуса соответственно равны 30 см и 24 см.

Тангенс угла наклона образующей к плоскости основания равен 2,4. Найдите тангенс

угла между плоскостью основания конуса и плоскостью сечения, проходящего через

вершину конуса и хорду АВ.

12. Площади основания усечѐнного конуса равны 25 см2 и 64 см

2, а площадь осевого

сечения равна 52 см2. Найдите образующую усечѐнного конуса.

13. Даны точки А( 32 ; - 1; 1) и В(0; - 3; 2). Укажите точку, принадлежащую сфере,

заданной уравнением 14222 zух .

14. Найдите радиус сферы, заданной уравнением .5,13 222 zуухх

15. Через точку В(3, 4; 12), принадлежащую сфере ,169222 zух проведена

плоскость, перпендикулярная оси OХ. Найдите расстояние от центра сферы до

плоскости сечения.

16. Радиус шара равен 13 см. Найдите длину линии пересечения сферы плоскостью,

проходящей на расстоянии 12 см от центра.

17. Все стороны квадрата касаются сферы диаметром 50 см, сторона

квадрата равна 14 см. Найдите расстояние от центра сферы до

плоскости квадрата.

18. Вершины прямоугольного треугольника с гипотенузой 24 см лежат на

сфере. Расстояние от центра сферы до плоскости треугольника равно

5 см. Найдите радиус сферы.

Page 58: 1. Пояснительная запискаjar.edunoskol.ru/images/dokument/RAB_PROG/geometria_coo.pdf · Стандарт основного общего образования

Ответы

к зачету № 3 по геометрии в 11 классе

по теме «Цилиндр, конус и шар»

№ задания

1 вариант

2 вариант

3 вариант

4 вариант

1 1 3 90о 90о

2 1 2 6 5

3 1 1 36 3

4 1 1 3 5

5 2 1 2 3

6 1 2 40 5

7 3 3 13 8

8 1 2 18 6

9 3 1 8 2

10 2 2 216о 10

11 1 2 1,2 4

12 3 3 2 5

13 3 2 В А

14 2 1 2 2

15 3 1 5 3

16 3 3 64 10

17 1 2 18 24

18 2 3 12 13

19 3 2

20 3 3

21 1 2

22 2 1

23 3 2

24 2 1

25 2 3

26 1 3

27 3 1

Геометрия 11 класс

Зачет № 4

по теме «Объѐмы тел»

Вариант 1

1. Какое утверждение неверное?

1). Если тело составлено из нескольких тел, то его объѐм равен сумме объѐмов этих тел.

2). Равные тела имеют равные объѐмы.

3). Если объѐмы тел равны, то тела равны.

2. Какое утверждение верное?

1) Не могут быть равны объѐмы четырѐхугольной призмы и четырѐхугольной пирамиды,

имеющих равные высоты.

2) Две призмы с равными высотами равновелики, если их основаниями являются

одноимѐнные многоугольники с равными сторонами.

3) Диагональные плоскости делят параллелепипед на равновеликие части.

3. Объем правильной шестиугольной призмы можно найти по формуле:

Page 59: 1. Пояснительная запискаjar.edunoskol.ru/images/dokument/RAB_PROG/geometria_coo.pdf · Стандарт основного общего образования

1) 325,0 2haV ; 2) haV 2 ; 3) 35,1 2hav ,

где a - сторона основания, h - высота призмы.

4. S – площадь боковой поверхности цилиндра, R – радиус его основания. Тогда объем

цилиндра равен:

1) RS; 2) 2

1RS; 3) 2RS.

5. Объем равностороннего конуса (длина образующей равна длине диаметра основания

конуса) нельзя вычислить по формуле:

1) ,9

1 3HV где H - высота конуса;

2) ,3

3 3RV где R - радиус основания конуса;

3) ,3

4 2 HlV где l - образующая конуса.

6. Объем усеченного конуса, высота которого равна H , вычисляется по формуле:

1) ,3

22121 RRRRHV где 1R и 2R - радиусы оснований;

2) ,3

1MNNMHV где M и N - площади оснований;

3) ,12

121

2

2

2

1 ddddHV где 1d и 2d - диаметры оснований.

7. Объем шара радиуса R можно найти по формуле:

1) 34 RV ; 2) 3

4

3RV ; 3) 3

3

4RV .

8. Площадь поверхности шара диаметром d можно найти по формуле:

1) 2dS ; 2)

22 dS ; 3) 2

4

3dS .

9. Имеются шар и куб равного объема. У какого тела больше полная поверхность:

1) у шара; 2) у куба; 3) площади поверхностей тел равны.

Геометрия 11 класс

Зачет № 4

по теме «Объѐмы тел»

Вариант 2

1. Какое утверждение верное:

1) Равные объемы могут иметь только равные тела.

2) Равновеликие тела – это тела, совмещаемые наложением.

3) Если первое тело содержит второе, то объем первого тела не меньше объема второго.

2. Какое утверждение верное:

1) Объемы двух правильных четырехугольных призм равны, если их диагональные

сечения равновелики.

2) Два прямоугольных параллелепипеда с разными измерениями имеют разные объемы.

Page 60: 1. Пояснительная запискаjar.edunoskol.ru/images/dokument/RAB_PROG/geometria_coo.pdf · Стандарт основного общего образования

3) Два прямоугольных параллелепипеда разных объемов не могут иметь одинаковые

измерения.

3. Объем правильной треугольной призмы можно найти по формуле:

1) 325,0 2haV ; 2) haV 2 ; 3) 35,1 2hav ,

где a - сторона основания, h - высота призмы.

4. Разверткой боковой поверхности цилиндра является квадрат со стороной a см. Тогда

объем цилиндра равен:

1) 34 ; 2) 4

3a; 3)

2

3a.

5. Объем конуса можно найти по формуле:

1) lRV 2

3

1 ; 2) 22

3

1HlHV ; 3) RHV

3

2 .

6. Объем усеченного конуса, высота которого равна H , вычисляется по формуле:

1) ,3

12121 RRRRHV где 1R и 2R - радиусы оснований;

2) ,3

1MNNMHV где M и N - площади оснований;

3) ,6

121

2

2

2

1 ddddHV где 1d и 2d - диаметры оснований.

7. Объем шара радиуса R можно найти по формуле:

1) 24 RV ; 2) 3

3

4RV ; 3) 3

4

3RV .

8. Площадь сферы радиуса R можно найти по формуле:

1) 2RS ; 2)

24 RS ; 3) 2

3

4RS .

9. Радиус шара уменьшили в 5 раз. Тогда площадь поверхности шара уменьшится:

1) в 2 раза; 2) в 10 раз; 3) в 25 раз.

Геометрия 11 класс

Зачет № 4

по теме «Объѐмы тел»

Вариант 3

1. Площадь полной поверхности куба равна 6 см2. Найдите его объем.

2. Диагональ правильной четырехугольной призмы равна 3, 5 см, а диагональ боковой

грани – 2,5 см. Найдите объем призмы.

3. Площадь сечения, проведенного параллельно оси цилиндра на расстоянии 4 см от нее,

равна 36 см2. Радиус основания цилиндра равен 5 см. Найдите его объем.

4. Диагональ квадрата, лежащего в основании правильной пирамиды, равна 8 дм, а ее

высота – 12 дм. Найдите объем пирамиды.

Page 61: 1. Пояснительная запискаjar.edunoskol.ru/images/dokument/RAB_PROG/geometria_coo.pdf · Стандарт основного общего образования

5. Радиус основания конуса равен 20 см, расстояние от центра основания до образующей

равно 12 см. найдите объем конуса.

6. В наклонной призме основание – прямоугольный треугольник. Гипотенуза которого

равна 4 см, один острый угол 30о. боковое ребро равно 5 см и составляет с плоскостью

основания угол 60о. Найдите объем призмы.

7. Сколько шариков диаметром 2 см можно отлить из металлического куба с ребром 4 см?

8. Площадь поверхности полушара равна 18 см2. Найдите объем шара.

Геометрия 11 класс

Зачет № 4

по теме «Объѐмы тел»

Вариант 4

1. Измерения прямоугольного параллелепипеда относятся как 2 : 3 : 4. Диагональ

параллелепипеда равна 29 см. Найдите объем параллелепипеда.

2. Основанием прямой призмы служит прямоугольный треугольник с катетами 6 см и 8

см. высота призмы равна радиусу вписанной в основание окружности. Найдите объем

призмы.

3. Площадь сечения, проведенного параллельно оси цилиндра на расстоянии 3 см от нее,

равна 64 см2. Радиус основания цилиндра равен 5 см. Найдите его объем.

4. Основанием пирамиды является прямоугольный треугольник с катетами 6 см и 8 см.

Каждое боковое ребро равно 13 см. Найдите объем пирамиды.

5. высота конуса равна 20 см, расстояние от центра основания до образующей равно

12 см. Найдите объем конуса.

6. в наклонной треугольной призме стороны основания равны 5 см, 6 см и 9 см. Боковое

ребро равно 10 см и составляет с плоскостью основания угол 45о. Найдите объем

призмы.

7. сколько кубиков с ребром 2 см можно отлить из металлического шара диаметром 4 см?

8. Объем шара равен 3

32 см

3. Найдите площадь поверхности полушара.

Геометрия 11 класс

Ответы к зачету № 4 по теме «Объѐмы тел»

задания

Вариант 1 Вариант 2 Вариант 3 Вариант 4

1 3 3 1 24

2 3 3 3 48

3 3 1 150 200

4 2 2 128 96

5 3 2 2000 1500

6 3 2 15 100

7 3 2 16 4

8 1 2 18 8

9 2 3

Page 62: 1. Пояснительная запискаjar.edunoskol.ru/images/dokument/RAB_PROG/geometria_coo.pdf · Стандарт основного общего образования

Геометрия 11 класс

Итоговый тест за курс 11 класса

Вариант 1

1. Выберите верное утверждение:

А) Векторы 𝑎 −5; 3; −1 и 𝑏 6; −10; −2 коллинеарны.

Б) Сечение конуса плоскостью, проходящей через его вершину, представляет собой

круг.

В) Объем цилиндра не изменится, если диаметр его основания увеличить в 2 раза, а

высоту уменьшить в 4 раза.

Г) Полый медный шар, диаметр которого равен 10 см, а толщина стенки 2 мм, будет

плавать в воде (плотность меди 8,9 г/см3).

Д) Радиус сферы 𝑥2 + 𝑦2 + 𝑧2 + 6𝑥 + 2𝑦 − 4𝑧 + 18 = 0 равен 2.

2. Даны три вектора, удовлетворяющие условию 𝑎 − 𝑏 − 𝑐 = 0 , 𝑎 = 3, 𝑏 = 4, 𝑐 = 5.

Вычислите 𝑏 ∙ 𝑐 − 𝑎 ∙ 𝑏 − 𝑐 ∙ 𝑎 . А) 25 Б) – 25 В) 50 Г) – 50 Д) 12

3. Площади граней прямоугольного параллелепипеда равны 5 см2, 10 см

2 и 2 см

2. Найдите

его объем.

А) 20 см3 Б) 16 см

3 В) 8 см

3 Г) 10 см

3 Д) Другой ответ

4. Радиус кругового сектора равен 6 см, а его угол равен 60о. Сектор свернут в коническую

поверхность. Найдите площадь основания конуса.

А) π см2 Б) 2π см

2 В)

𝜋

2 см

2 Г)

𝜋

3 см

2 Д)

𝜋

6 см

2

5. Найдите объем треугольной пирамиды, боковые ребра которой взаимно

перпендикулярны и равны соответственно 4 см, 5 см и 6 см.

А) 20 см3 Б) 40 см

3 В) 120 см

3 Г) 60 см

3 Д) 10 см

3

6. Угол при вершине осевого сечения конуса равен 2φ. Периметр осевого сечения равен

2р. Найдите объем конуса.

А) 𝜋𝑝3 sin 2𝜑

6; Б)

𝜋𝑝3 sin 2𝜑 sin 𝜑

6(1+sin 𝜑)2; В)

𝜋𝑝3 sin 2𝜑 sin 𝜑

3(1+sin 𝜑)3;

Г) 𝜋𝑝3 sin 2𝜑 cos 𝜑

6(1+sin 𝜑)3; Д)

𝜋𝑝3 sin 2𝜑 sin 𝜑

6(1+sin 𝜑)3.

7. В цилиндр вписан правильный тетраэдр со стороной 3. Найдите объем цилиндра.

А) 2𝜋; Б) 𝜋 2; В) 𝜋 3; Г) 𝜋; Д) Определить нельзя

8. В треугольнике АВС: А(0; 0; 0), В(2; -1; 3), С(-1; 1; 1). Найдите диаметр окружности,

описанной около этого треугольника.

А) Определить нельзя; Б) 17; В) 14; Г) 3; Д) 13.

9. Плоскость пересекает шар. Диаметр шара, проведенный в одну из точек линии

пересечения, равен 4 3 см. Найдите угол между диаметром и плоскостью сечения,

если площадь сечения равна 6𝜋 см2.

А) 60о; Б) 120

о; В) 30

о; Г) 45

о; Д) 90

о.

Page 63: 1. Пояснительная запискаjar.edunoskol.ru/images/dokument/RAB_PROG/geometria_coo.pdf · Стандарт основного общего образования

10. Найдите объем полого шара, если радиусы его внутренней и внешней поверхности

равны 3 см и 6 см.

А) 126 𝜋 см3; Б) 189 𝜋 см

3; В) 252 𝜋 см

3; Г) 315 𝜋 см

3; Д) 378 𝜋 см

3.

Геометрия 11 класс

Итоговый тест за курс 11 класса

Вариант 2

1. Выберите неверное утверждение:

А) Векторы 𝑎 3; −5; −1 и 𝑏 6; −10; −2 коллинеарны.

Б) Сечение цилиндра плоскостью, проходящей через его ось, представляет собой

прямоугольник.

В) Площадь сферы увеличится в 4 раза, если ее радиус увеличить в 2 раза.

Г) Конический бак, имеющий глубину 3 м и радиус круглого верха 1,5 м, вмещает не

более 5000 л воды.

Д) Радиус сферы 𝑥2 + 𝑦2 + 𝑧2 − 6𝑥 − 2𝑦 + 4𝑧 + 5 = 0 равен 3.

2. Даны три вектора, удовлетворяющие условию 𝑎 + 𝑏 + 𝑐 = 0 , 𝑎 = 1, 𝑏 = 4, 𝑐 = 5.

Вычислите 𝑏 ∙ 𝑐 + 𝑎 ∙ 𝑏 + 𝑐 ∙ 𝑎 . А) 21 Б) – 21 В) 42 Г) – 42 Д) 10

3. Площади граней прямоугольного параллелепипеда равны 6 см2, 3 см

2 и 2 см

2. Найдите

его объем.

А) 3 см3 Б) 4 см

3 В) 6 см

3 Г) 11 см

3 Д) Другой ответ

4. Радиус кругового сектора равен 8 см, а его угол равен 45о. Сектор свернут в коническую

поверхность. Найдите площадь основания конуса.

А) π см2 Б) 2π см

2 В) 4𝜋 см

2 Г)

𝜋

2 см

2 Д)

𝜋

4 см

2

5. Найдите объем треугольной пирамиды, боковые ребра которой взаимно

перпендикулярны и равны соответственно 2 см, 4 см и 6 см.

А) 24 см3 Б) 16 см

3 В) 15 см

3 Г) 10 см

3 Д) 8 см

3

6. Угол при основании осевого сечения конуса равен 2φ. Периметр осевого сечения равен

2р. Найдите объем конуса.

А) 𝜋𝑝3 sin 𝜑

12; Б)

𝜋𝑝3с𝑜𝑠22𝜑 sin 𝜑

64𝑐𝑜𝑠 5𝜑; В)

𝜋𝑝3 cos 2 2𝜑 sin 𝜑

12𝑐𝑜𝑠 5𝜑;

Г) 𝜋𝑝3 cos 22φsin 𝜑

24𝑐𝑜𝑠 5𝜑; Д)

𝜋𝑝2 cos 22φsin 𝜑

12𝑐𝑜𝑠 5𝜑.

7. В цилиндр вписан куб со стороной 3. Найдите объем цилиндра.

А) 𝜋 6

2; Б)

𝜋 3

2; В)

3𝜋 6

2; Г)

3𝜋 3

2; Д) Определить нельзя

8. В треугольнике АВС: А(0; 0; 0), В(1; 2; 1), С(1; -1; 1). Найдите координаты центра

окружности, описанной около этого треугольника.

А) (1; 0,5; 1); Б) (0,5; 1; 0,5); В) (0,5; -0,5; 1); Г) (2; 1; 2); Д) Определить нельзя

Page 64: 1. Пояснительная запискаjar.edunoskol.ru/images/dokument/RAB_PROG/geometria_coo.pdf · Стандарт основного общего образования

9. Плоскость пересекает сферу. Диаметр сферы, проведенный в одну из точек линии

пересечения, равен 4 2 см. Найдите угол между диаметром и плоскостью сечения,

если длина линии пересечения равна 4𝜋 см.

А) 30о; Б) 45

о; В) 60

о; Г) 90

о; Д) 120

о.

10. Найдите объем полого шара, если радиусы его внутренней и внешней поверхности

равны 10 см и 7 см.

А) 438 𝜋 см3; Б) 1792 𝜋 см

3; В) 800 𝜋 см

3; Г) 615 𝜋 см

3; Д) 876 𝜋 см

3.

Ответы к итоговому тесту по геометрии за 11 класс

Задания 1 2 3 4 5 6 7 8 9 10

Вариант 1 В Б Г А А Д Б Б Г В

2 Г Б В А Д В Г А Б Д

6. Перечень учебно-методических средств обучения

Программа: Программы общеобразовательных учреждений: Геометрия. 10–11 классы /

Сост. Т.А.Бурмистрова. – М.: Просвещение, 2009.

Учебник: Геометрия, 10–11: Учебник для общеобразовательных учреждений /

Л.С.Атанасян, В.Ф.Бутузов, С.Б.Кадомцев и др. – М.: Просвещение, 2002.

Дидакт. материалы: Зиб Б.Г. Геометрия: Дидактические материалы для 10 кл./ – М.:

Просвещение, 2012.

Демонстрационный вариант контрольных измерительных материалов

ЕГЭ-2014 года по математике / ФИПИ

О б о р у д о в а н и е

1.Таблицы по геометрии.

2. Набор по стереометрии:

многогранники:

Призма

Прямая призма

Пирамида

Правильная пирамида

Правильная усечѐнная пирамида;

тела вращения:

Page 65: 1. Пояснительная запискаjar.edunoskol.ru/images/dokument/RAB_PROG/geometria_coo.pdf · Стандарт основного общего образования

Цилиндр

Конус

Усечѐнный конус

Площадь сферы

Объѐм шара

3. Чертѐжные инструменты

Циркуль

Треугольник

Линейка

Транспортир